Download as pdf or txt
Download as pdf or txt
You are on page 1of 172

Littoto's

Problornutieal,
Reoreutioros
Compiled by
Jarnes tr'. Hurley
University of California
Riverside, Calilornia

VAN NOSTRAND REINHOLD COlllPANY


NeuYork Cincinnati Toronto London Melbourne
'r-
I
I

I
i

Prefaoe

During the past several years Litton Industries has sponsored


a series of mathem atical puzzlers called "Problematieal
Van Nostrand Reinhold Company Regional Offices: Recreations." The series has developed a large and enthusiastic
New York Cincinnati Chicago Millbrae Dallas
following not only among readers of the trade publications in
Van Nostrand Reinhold Company International Offices: which the Problematical Recreations originally appear but also
London Toronto Melbourne among the many others, particularly in Southern California,
fortunate enough to come upon some of the short booklets dis-
tributed by Litton which collect a few of the series togelher. The
Copyright O 1971 by LITTON EDUCATIONAL PUBLISHING, present work is intended to put together in zi single volume
INC. many of the best of the nearly 600 Problematical Recreations
Library of Congress Catalog Card Number:. 7l-161.487 which have appeared since the series began.
The pleasure and stimulation found in grappling wilh rnathe-
matical puzzles is not new, as is well attested to by the large
All rights reserved.. No parr of thi-s work c-overed b-y the copyright number of popular books on the subject. What has nade the
hereoi may be reproduied or used in any for-m or by any means- Problematical Recreations series so popular, though,has been
graphic, elictronii, or mechanical, including photocopying, recording, a certain spirit, hard to characterize perhaps, but certainly a
7"pi"g, or information storage and retrieval systems-without writ-
ten permission o! the publisher. reflection of the freshness and vibrancy of the problerts them-
selves. Many are highly original contributions submitted by
Manufactured in the United States of America fans of the series whose enthusiasm for it seems to have the
happy facility of being at once a product and a cause of its
Published by Van Nostrand Reinhold Company vitality. A prime goal in the selection of problems for the present
450 W. 33rd Street, New York, N. Y. 10001 collection has been capturing as much as possible ttre flavor
and spirit of the series as just described. Naturally, any editor
PUBLISHED SIMULTANEOUSLY IN CANADA BY choosing from a large collection of interesting problerns will
VAN NOSTRAND REINHOLD LTD"
to some extent be influenced by his own tastes and prejudices
in making his decisions. To those fans who would have liked
t5 14 13 12 11 10 9 8 7 6 5 4 3 2 1 to have seen more of their particular favorites included, apol-
ogies are in order. It is to be hoped, however, that theproblems
1-

presented here include a sufficiently comprehensive cross-sec-


iioo ro that no one will feel too disappointed.
Problems with the most general appeal were searched out
for the first two chapters. These required the least amount of
mathematical training and knowledge. Their main requirement
Oontents
is some insight, a little ingenuity, and most of all, a delight in
doing some mental gymnastics. Chapters 3 through 7 include
problems which involve some acquaintance with combinations
and probability, elementary algebra, Euclidean geometry, and
n.,*b"t theory. One does not by any means need to be a mathe- Prelace
matician to enjoy these problems, however, because here too
the best weapon is not mathematical knowledge but good old-
fashioned ingenuity. The last chapter includes problems best I For the Pereeptioe
attacked by those with some knowledge of college level mathe-
matics, mainly calculus. Problems of a general nature whose
Answers are provided for all the problems in the book. Many
solutions require insight as the prin-
are completely worked out in detail, but those who prefer just
being pointed in the right direction will be pleased to find many
cipal ingredientl
urr*w"i* in the form of generous hints. Finally, for the'very
independent who merely want to check their results, a few qD
unswet* are only that-answers whose getting to is left for the -f
For Sleuth* BZ
solver to devise.
Problerns requiring careful analysis
Problems marked by an asterisk appeared originally in
Mathematical Bafilers by Angela Dunn, copyright @ 1964 by and logical deduction.
McGraw-Hill, Inc., and are used by permission of McGraw-
HillBook ComPanY.
It is a pleasure to acknowledge the help and encouragement 3 For Ga,rnblers a7
of Mr. Michael J. Hamilton of Van Nostrand Reinhold in com-
piling this book. My wife Cecile is deserving of special thanks Problems involving probability and
ior hir assistance in cataloging the problems selected' determination of the number of out'
cornes of various situations.
JAMES F. HURLEY

O of C alil ornia
niv ersity 4 For the Atgebra.iea,llg
Riverside, Calilornia
I
fnclined r2s
February, 197
Problems best attacked by means of
variables and equations.
v1
vIl
il
i
i

.)- For the Geontetricallg


fnelined. l6s

Prohlems in-volving plane geometry


and trigonometry.

6 For Fans ol ['oB


Diopha.nttts THE PMBCEPTIVE
Problems involving DioPhantine
equations, those for which only whole
number solutions are sought'
Pt'ohletns
7 For Nurnber Theorists 22'D
of a ge?oeral ruat.,ore
Problerns involving properties of
integers.
uhose soltotiotts
reqe,Jiye im,si,gh,t a,a the
B Dlore Ad'oa'nced
Problerns
pvi?t eipal in gr edierot
.A,collection of problems from va'
rious fields which require calculus
or other sophisticated mathematical
tools.

Ansuters to Problerns

vlll
CHAPTER 1

PR 30
Rufus T. Flypaper drives two miles to work every morning- Very
precise, he knows he must average 30 mph to arrive on time. One
morning a woman driver impedes him for the first mile, cutting his
average to only l5 mph. He quickly calculated his proper speed for
the rest of his trip to arrive on time. Assume that his car could do
120 mph. Could he arrive on time? *California Engineer

Answer: No, he has already used 4 minutes


il-

CHAPTER 1
CHAPTER 1

46 47
Stations A and B are 120 miles apart on a single-track railroad' At
A hunter wished to take his one-piece rifle on a train but the con- the same time that a train leaves A for B at 25 mph, a train leaves
;;;. refused to permit it in the coach and the baggage man could
B for A at l5 mph. Just as the first train leaves A, a South American
not take any article whose greatest dimension exceeded I
yard. The
botfly flies from the front of the engine straight toward the other
i"ngth of the rifle was 1.7 yards' What could the hunter do?
train at IOO mph. On meeting the second train it immediately turns
back and flies straight for the first train. It continues to fly back and
forth with undiminished speed until it is crushed in the eventual
Answer: He could put his gun diagonally in a cubical
collision. How far had the fly flown ? -Ol ancient Hindu origin
box, 1 yard on a side

Answer: 300 miles

4
CHAPTER 1 CHAPTER 1

68
53
A man walks one mile south, one mile west, then one mile north, In the game of "stogeyl' two players alteri-rately place cigars on
gnding where he began. From how many points on the surface of a rectangular table with the restriction that each new cigar must
tne eartn can such a journey be made? (fhere are more than 1)' not touch any of the previously placed cigars- Can the 1 st player
-Contibuted assure himself of victory if we define the loser as the first player
who finds himself without sufficient room to place a cigar?
- Contri buted
CHAPTER 1
CHAPTER 1

69 98
Seven of the bricks weigh the same Maynard's Grandfather Clock is driven by two weights, one for
A bricklayer has 8 bricks- the striking mechanism which strikes the hours only' the other for
amount and 1 is a little heavier than the others' If the man has a
the time mechanism. When he hears the clock strike his bedtime'
balance scale how can he find the heaviest brick in only
2 weigh-
he immediately winds the clock and retires. After winding, the
ings?* -Contributed
weights are exactly opposite each other. The weights are again
opposite every six hours thereafter. What is Maynard's bedtime?
d
-Contribute

Answer: 9 pm or 3 am

[i
--'
q

i
1

CHAPTER 1 CHAPTEB 1

132 135
The game of reverse tic-tac-toe (known to some as toe-tac-tic) has Using graph paper to simulate a board of 64 squares and starting
the same rules as the standard game with one exception" The first anywhere, is it possible to move a Knight to all iquares without
player with three markers in a row loses. Can the player with the touching the same square twice? Move can be made lrorn A to
* either B. XXA
first move avoid being beaten? - Contibuted BXX
XBX - Contibuted

to t1
I
r-
LI

ri

It
CHAPTER 1

lo
CHAPTER 1

tI
ltli

lli

fr

ir'

l,
L

ilt
lil
li
l;

iii
i,l
142
1i
139
i mathematician' was shop- Find 1000 consecutive nonprime numbers. 't - Contributed
I Dr. Furbisher LaRouche, the noted
tl ping ut a harclware store and asked the
price of certain articles'
"One would cost 10 cents' eight would
ii iiJ .ut"**un replied, 20 cents' one hundred and
cost 10 cents, seventeen woul<I cost
Illl and fifty six would also
io". *o"fA cost 30 cents, seven hundred
seventy two would cost
' !'r
i
;. ..ti :O cents, antl one thousand and buying?*
+O ."ntr:' What was Dr' LaRouche -Contributed

12
i
I
l3
#ecr-
{-
i
il

CHAPTER 1
CHAPTER 1

TD IB

146 -i^168 , .. .r, .{ll r*J


SlA,s''suming the sun rises at 6:b0 a.m., sets at 6:00p.m., and moves
corners of a checker-
Two squares are removed from opposite at a uniform rate. how can a lost boy scout determine south by
be fllled with
board leaving 62 squares' Can the checkerboard means of a watch on a cloudless day? Contrihuted
3]'dominoes,eacht.lominocoveringtwoadjacentsquares?
-
-Contributed

l'ild\Iffi11

Answer: Align the hour hand with the sun's azimuth,


and south will be midway between the hour hand and
12.

t5
l4
CHAPTER 1 CHAPTER 1

l1 l5

206
%w*
174
#
What letter f ollows OTT FFSSE-?
s[
*-
- Contributed The numbers are divided into three groups as follows: 0,3,6,8'9,
.".in the first group, 1,4,7,11,14,...in the second group and
2,5,10,12,13,.. . in the third. In which groups would 15,16 and
17 be placed? *
-Contributed

Answer: 15 and 16 would be in the third group


17 in the second group

l6 t7
CHAPTER 1

t7
CHAPTER 1

t6

274
225
Determine the next three terms of the sequence 12, 1, 1, !,"-'
pitcher Hi N' Outside
fn a fast Major League baseball game'number of pitches possible'
-Contributed
managed to get by wittritte- minimum prior to com-
ii"'no*u tie e.,tire game, which was not called
;;il;:Ho* *u.,v pithes ditl he make? -contributed Answer: 2, 1 and 3

Answer: 25

19
18
CHAPTER 1

t9
CHAPTEB 1

IE

be disengaged from The set A contains the integers O,4,5,9,11,12,13,14,19, ' ' ' '
What is the least number of links that must
a 23-link chain so that any number of
links from I to 23 can be The set B contains 1,2,3,6,7,8,10,15,16,17,'18, ' " ' Place 20 and
obtained by taking one or more of the pieces? - contributed 21 in their proper sets. -Contributed

Answer: 2 the 4th and 11th

2l
2A
;-

CHAPTER 1
CHAPTER

2t
1

20
t?
7

football game of the season 17 A chemist has three large test tubes and a beaker with 54 c.c. of
State University won their first elixir. Using the test tubes and ingenuity only, how cari he retain
they managed to score more
to 0. Though they scored no safeties' in the previous quarter' 50 c.c. in the beaker?
;;"fi qu..i.. than they had scored - Contribated
;;,t scores? Contributed
iltro, *.r" Stut"'t quarter -

Answer: 0, 3, 6, and 8

ar, i 23
&
r-&!_
T

CHAPTER 1
CHAPTER 1

O),5rt,
-et
28

=Gql
Using only the above symbols
Gir"-you.i"lf full credit if you
three valid
get one'
equations are possible"
- Contributed
How many qolors are necessary for the squares of a chessboard
in order to assure that a bishop cannot move from one square to
Snother of the same color? - Contributed

Answer: 8 colors

:.
r$

24 25
s .

,tca-*
t.
ll Y
i
i

CHAPTER 1
CHAPTER 1

21 26

h.

The passengers on an excursion bus consisted of 14 married


A neat computer programmer wears a clean shirt every day' If couples, 8 of whom brought no children, and 6 of whom brought
h" d.op. o[ his laundry and picks up the previous week's load 3_children apiece. Counting the driver, the bus had 31 occupants.
ftlonaay night, how many shirts must he own to keep him How is this possible?
"u".y
going? - Contributed contibuted

Answer: 7 shirts Answer: Included among the 18 children were 8 married


couples.

tu
26
ffi%-- 2t
CHAPTER 1 CHAPTER 1

26 27

Very few people are aware of the growth pattern of Jack's bean- Tom, Dick, and Harry played a round of golf, each ending with
stalk. On the first day it increased its height by Vz, on the second a total of 72 strokes. Each pair competed against each other in
day by tA, on the third day by Vq, and so on' How long did it take match play (most holes won). Tom beat Dick, and Dick beat
to achieve its maximum height (100 times its original height)? Harry. Does it follow that Torn beat Harry?
-Contibuted
-Contributed

Answer 198 days

ar

29
2A ffin
CHAPTER 1
CHAPTER 1

2E Dg

Show, with a simple example, that an irrational .number raised to


Six boys on a hockey team pick a captain by forming a circle an trrational power need not be irrational.
and counting out until only one remains' Joe is given the option -CorLtributed
of deciding number to count by. If he is second in the original
ihat
*Contributed
counting o-rder what number should he choose?

Answer: He should choose 10

30
3l
fi
T

lt

sl
CHAPTER 1
CHAPTER:I

ao

ii

cities of Puevigi and their road con-


A jig-saw puzzle contains 100 pieces. A "move" cdnsists of con-
Depicted above are the 22 necting two clusters (including "clusters" of just one piece.) What
tour which visits each citY
nections. Can You devise a continuous is the minimum number of moves required to complete t2Zny;:)il
exactlY once? -Contributed

Answer: 99 moves

32 .';}
M
tr
$

CHAPTER 1
GHAPTER 1

s2 BB
a aaaaa
+ a
a a a

itl
+: a
a l+
s a
atata
a
a
a
a

+
In the game of "connecto", 2 players alternate in joining adjacent
statement of this problem' You
must'
There are three errers in the points, horizontally or vertically, on an infinite rectangular lattice,
detect all of them t""i"'" full credit' -Contributed' one using solid lines for his connections, the other, dashes. The
' winner is the first to enclose a region of any shape by a boundary
composed of his symbol only. (The player with the dashes has won
above). Is the 2nd player doomed to defeat? - Conrributed

.l
I

ri,
r'il
rll
.i
,rll
ji
tir
ffi
,ffi
ffilr. 35
rir 34
iir
lll
CHAPTER 1
CHAPTER 1

B1 B5

"Four" in English, "cinco" in Spanish, and "ni" and "san" in


volume set of encyclopedias
A novice librarian shelved a twelve Japanese share an interesting property. What is il? - Contributed
order ito* t"f' to right' Volumes 8' l1' 5' 4' 9'
in the following j' syitem' where will the annual
1,7 , 6, lO, 3, 12, ",a Uti"g her
go? Contributed
s.lppf"i*.,t, Volume 13,

Answer: Between volumes 10 and 3.

,4
dt
36
jtif,I-

CHAPTER 1
CHAPTER 1

86 87

At the age of 17, Gauss proved that a regular polygon of 17


White to move and mate in one move! -Contributed sides can be constructed with ruler and compasses. Suppose
every side and every diagonal is painted either red, white or
blue- Prove at least one triangle is formed with all three sides
Painted the same color.
-Conlributed.

38 39
ilr-

CHAPTER 1
CHAPTER 1

BE B9

B\''l
l,i'i
.:;
t-/

In Greenwich Village, tic-tac-toe is played in an unusual way-


The League Against Restrictive Diets, with members all over At each turn a player marks as many squares as he wishes pro-
the U.S.; plans a convention. Most of the members live in vided they are in the same vertical or horizontal row (they need
Chicago, tb tt feel that city is the logical site' The other
"ysome city representing the "weighted centroid" not be adjacent). The winner is the one who marks the last
memblrs suggest square. Which player has the aclvantage and what strategy
of the League If the object is to minimize total distance traveled should he employ?
by the -"^b".t of the L.A.R.D'' who is right? ---Contributed -Contributed

Answer: Chicago

40 4t
I
!
1

CHAPTER 1 CHAPTER 1

10 1T

With some sharp reasoning, you ought to be able to determine Archimedes O'Toole, a mathematical poet, on seeing this equa'
the last member of the sequence for which the first 20 mem- tion, translated it into a limerick. Can you duplicate this feat?
bers are: 11, 31, 71,91,32,92, 73, 73, 14, 34, 74, 35' 95, 16, 76,
77, 37, 97, 38, 98, _?
Sltt#i-g+ s (11) : e,t o" -Contributed
-Contributed

Answer: 79

42 43
CHAPTER 1 CHAPTER 1

12 18

Wgrre, ro PLAY AND MATE IN Two MovES' Neither your rook There are at least two ways of representing 20, using three 3's
nor king has previously moved. Following your announced mate and standard mathematical symbols. Find one.
-Contributed
in two moves, your opponent, black, offered to bet $1000 that it
was impossible. You accepted with pleasure' Your move'

i
t
I
M I 45
".#e*--*.
I
I
i

$ii
t,
ll

CHAPTER 1
CHAPTER 1

11 15
i;

Ii
E:

I,

A simple substitution cipher message was worked out on a black-


What operation can be perfgrmed three successive tinles on a solid board and accidentally erased. A few fragments remain, how-
cube, so that at each stage, the surface area is reduced in the same ever. The word G Q K X Y J has escaped erasure with X iden-
proportion as the volume? -lontributed tified as R and Y as B. Also the word P K Z X D V can be
made out with Z identified as L. The only other legible vord is
K v J Z D c. what word does this represent?
a onfiibuted

46 47
CHAPTER 1
CHAPTER 1

16 17
'i- [
fi\ fi\ -/\rt Ft

(.vl {a7*xr,
lri
ll
ii
h,-
I
I

A safe has three dials shown above. It will open only when a
The following pairs are members of a certain relation: (Sacra-
i
three-letter word is indicated by the dials even in permuted form.
*"lt9l ,Carson City), (Pierre, Bismark), (Juneau, Otympia),
I

by L. l. Upton and (Albany, Hartford). Moreover, the reversals of the firsf two
rl What is that word? pairs are also members, while those of the latter two are not.
tl -Contributed
'- What is the relation? by Paul Reisinger
-Contributed

4B 49
CHAPTER 1 GHAPTER 1

1E 19

A lighthouse shows successive one-second flashes of red, white, Wink your right eye, and your mirror image winks its lelt' Hold
green, gree.r, white, red. A second lighthouse does the same only out your left hand, and the image holds out its right. Since the
with twt-re"ond flashes. The six-second sequence of the first light- mirror reverses everything in the horizontal direction, why not
house is repeated steadily, as is the twelve-second sequence of the vertical? For example, why doesn't the mirror show yotl
the other lighthouse. What fraction of the time do the two lights standing on your head? by David L' Silverman
show the same co.lor if the given sequences start at the same -Contributed
time? -lcsntributed by Otis N. Minot

Answer: 1/6

50 5l
CHAPTER 1 CHAPTER 1

50 5t

E:.-.-
-='--

*12'3
Four players played a hand of hearts at $1 a point (pairwise 4567," I thought. "I'll woo my lady fair
payoffi;.-Dave lost $10 to Arch, $12 to Bob, and $20 to Chuck' With 6154"723." AIas, at greater cost
How many hearts did Poor Dave take My rival (6147 3521) staked out his claim
)iu"r"o bv c. w.Gardner _ With orchids dear to maidens' hearts. 1,5'l 45321
There are four anagrams to decipher in this cryptogram.
35467 12! by H. Rosenberg
-(6n1yifiy1ett

52 53
ii
CHAPTER 1 CHAPTER 1

52 58
THEREARE 3 frAYs,l,l
1']O SPEI.ITHE WORD

"ABCD EF FG HCHI CF C JCK EL MNLI?" an appren- The student above can't decide whether to write "to," "tlo," or
tice poetician asked Archimedes O'Toole. (Question enciphered "two". In point of fact, the sentence can be spoken but not writ-
by simple substitution.) "How about February 29th for openers?" ten. Can you give an example of a sentence that can be written
retorted Arch. What was the apprentice's question? -- but not spoken? Contributed by Ben Rogers
by David L. Silverman -
-Contributed

i
I

I
I
54 i.
,s@,",
55
;ry!,,.
l-_

CHAPTER 1

51
X'OB SLMUTHS

Problerns requirimg
' earef?Ll um,aly sis and
logieul dedoootiort
In a memorable game with the Podunk Polecats, the Mudville
Mets established a record. They received the maximum number
of walks possible in one inning in which one player (who hap-
pened to be the Mighty Casey) was up three times and accounted
for all three outs. How many walks did Podunk allow in that
tedious half-inning? -Contributed by C.ll. Gardner

Answer: 30 walks

50 57
CHAPTER 2

Mary Ann Moore's father has a yacht and so has each of his four
friends: Colonel Downing, Mr. Hall, Sir Barnacle Hood, and Dr.
Parker. Each of the five also has one daughter and each has named
'\is yacht after a daughter of one of the others. Sir Barnacle's
yacht is the Gabrielle, Mr. Moore owns the Lorna; Mr. Hall the
Rosalind. The Melissa, owned by Colonel Downing, is named af-
ter Sir Barnacle's daughter. Gabrielle's father owns the yacht
which is named after Dr. Parker's daughter. Who is Lorna's father?
-Kansas State Engineer

Answer: Colonel Downing

59
I
I

CHAPTER 2 CHAPTER 2

A Solid State Physicist gives a small stag party. He invites his Assume that every tree has at least one leaf. If there are more trees
father's brother-in-law, his brother's father-inlaw, his father-in- than lhere are leaves on any one tree, then there exist at least two
law's brother, and his brother-in-law's father. Find the number of trees with the same number of leaves. Is the conclusion valid?
guests" -School Science & Malhemalics
-ailapted lrom Charles Lutwidge Dodgson

60 6l
CHAPTER 2
CHAPTER 2 il

iil

il

itl

I
I

j:

.i
I

I
ll

i
i

A college president, a professor, an instructor, and a janitor are


A rich farmer had l5 children by his first wife and 15 by his second. named Mr. Brown, Mr. Green, Mr. White, and Mr. Black, but not
The second wife wished to insure the heritage to one of her own respectively. Four students with the same names will be designated
children and persuaded him to seat all 30 in a circle and count off here as Brown, Creen, White and Black. The student with the same
every tenth child until only I remained. The first l4 thus eliminated -na-e as the professor belongs to Black's fraternity. Mr. Creen's
were all his first wife's children. From this point on he insisted that daughter-in-law lives in Philadelphia" The father of one of the stu-
they count backward from his first wife's lone remaining child. In dents always confuses White and Green in class, but is not absent-
what order were his first wife's children? Who became heir? minded. The janitor's wife has never seen Mr. Black. Mr. White is
-Colilornia Engineer the instructor's father-in-law and has no grandchildren. The presi-
dent's oldest son is seven. What are the names of the president,
professor, instructor, and janitor?
bY a Recrealioner
-Contributed

Answer: Mr White - President, Mr. Brown -


Professor; Mr. Black - Instructor, Mr. Green
Janitor

63
62
rii

il
i-
I

CHAPTER 2 CHAPTER 2

On a certain campus liberal arts students always lie and engineers Three men are blindfolded in a mirrorless room. Each is told that
always tell the truth. A stranger meets 3 students and asks the first he may or rnay not be given a mark on his forehead, and is
if he is studying liberal arts. The first answers the question, but the instructed that when the blindfolds are removed each is to raise
stranger doesn't hear him. The second student then says that the first his hand if he sees one or more marked foreheads, and to lower
denied being a liberal arts student. Then the third student says that his hand when he rationally concludes whether his own forehead
the first is really a liberal arts student. How many are Iiberal arts is or is not marketl. Unknown to the participants, all foreheads
students? Can we decide which? -01 ancient origin are actually marked, The blindfolds are removed, and alt hands
are instantly raised. One of the men shortly lowers his hand' By
what logical process does he know he is marked?
- A merican M a t h e matical M ont hlY
Answer: There is only one liberal arts students.

64 o5
CHAPTER 2
CHAPTER 2

A canoe is floating in a swimming pool. Which will raise the level


Four boys, Alan, Brian, Charles and Donald, and four girls, Eve,
of the water in the pool higher, dropping a penny into the pool
Fay, Gwen and Helen are each in love with one of the others, and,
or into the canoe? Or does it make any difference? *
sad to say, in no case is their love requited. Alan loves the girl who Contributed
loves the man who loves Eve. Fay is loved by the man who is
-
loved by the girl loved by Brian. Charles loves the girl who loves
Donald. If Brian is not loved by Gwen, and the boy who is loved
by Helen does not love Gwen, who loves Alan?
* -Contibuted Answer: Dropping the penny into the canoe will
raise the water level higher

Answer: Gwen loves Alan

67
66
tt
CHAPTER 2 CHAPTER 2

IO

Five suspects were rounded up in connection with the famous In any gathering of six people prove that either three are rnutually
"Cock Robin Murderl' Their statements were as follows: A: acquainted or three are mutually unacquainted. -Contributed
"C and D are lyingl'B: "A and E are lyingl'C: "B and D are
lyingl' D: "C and E are lyingl' E: "A and B are lyingl'Who is
lying? * -Contrihuted

Answer: A is lying

6B 69
CHAPTER 2
CHAPTER 2

tp IB

Six grocers in a town each sell a different brand of tea in four Between Kroflite and Beeline are five other towns. The seven
ounce packets at 25 cents per packet. One of the grocers gives towns are an integral number of miles from each other along a
short weight, each packet of his brand weighing only 37a ounces. straight road. The towns are so spaced that if one knows the
If I can use a balance for only one weighing, what is the number of miles a person has traveled between any two towns
he can determine the particular towns uniquely. What is the mini-
minimum amount I must spend to be sure of finding the grocer
who gives short weight?'lt mum distance between Kroflite and Beeline to make this possible?t
-Contrihuted
- Contibuted
Answer: At least 25 miles
Answer: 3.7 dollars

70 7l
CHAPTEB 2 CHAPTER 2

t1 t5

A man has red, gray and black flagstones for making a walk- He In the game "subtract-a-square," a positive integer is written down
wants no two consecutive stones to be the same color, no con- and two players alternately subtract squares from it with the
secutive pair of stones to have the same two colors in the same restriction that the remainder must never be Iess than zero.
order, no repetition of three consecutive colors' etc. He starts out -The player who leaves zero wins. What square should the first
laying first a red stone, then a gray, and continues until he finishes player subtract if the original number is 29? * - Contributed
laying the seventh stone. He then finds himself stymied and un-
able to use any stone for the eighth without repetition of some
color pattern. What were the colors of the first seven stones?*
- Contributed

n, 73
:rc-.
le."- "
CHAPTER 2 CHAPTER 2

t6 t7

There are five weights, no two weighing the same' With a beam Strephon and Phyllis decide to test their love with a daisy. They
balance, arrange the weights in order from heaviest to lightest in agree to pluck petals alternately, taking either one petal or two
seven weighings-
* -Contributed adjacent petals. There are 13 petals altogether. He picks one
saying, "She loves mel' She picks two adjacent petals, leaving
two groups of 8 and 2, saying, "He loves me notl' How should
Strephon continue? * -C0ntibuted.

74 i I;I
"$&
f-
l

CHAPTER 2
CHAPTER 2

IE r9

A game proceeds as follows: Smith starts with a l0-link chain and


Alamphasthreeon-offbuttons,allofwhichnrustbeonforthe removes any single link, presenting Jones either with a 9-link
lamp to light. A man wishes to turn on the lamp at a moment chain or two chains having a total of 9 links. Jones removes any
when only the second switch is off. He does not know this and
--one link from each of the chains. Smith removes any one link
proceeds to press the first hutton. Getting no result he presses the
from each of the remaining chains and so on, until the winner
iecond and eventually, on the seventh trial' never repeating any
(
removes the last link or links. What should Smith dq first?
on-off configuration), the Iamp finally lights' In what order did he
press the buttons'l
*C orttribute il -Contributed

Answer: Smith should leave two chains of 6 and 3


Answer: 1,2,3,2,1,2,3
links

lt
76
T-
I
I

I
I

I
l

CHAPTER 2 CHAPTER 2

20 2l

ffi
In the country of Puevigi, the population consists of Soothsayers, What property is common to Arctic penguins, peacock eggs, the
who never lie, Dissemblers, who always lie, and Diplomats, who Hungarian Merchant Marine, the University of Chicago football
alternately lie and tell the truth. If you meet a citizen of Puevigi, team, 19 point cribbage hands and the solution set of the equation
how with just two questions can you determine to which group -ge": 1?
- Contributed
he belongs? -Contributed

7B 79
I

cl-tAPTER 2 CHAPTER 2

ED,qD,
_a PB

- ...:
.--\

In Bristol 90% of the citizens drink tea; 80% drink coffee;7O%o


Prove that at least two Irishmen have the same number of Irish
drink whiskey; and 6O7o drink gin. No one drinks all four bev-
friends. -Contributed erages. What percent of Bristol's citizens drink liquor?
*Conrributed

Answer: 100%

t
80 8t
*
T
I

CHAPTER 2 CHAPTER 2

21 25

.:t

A teenager wants to go out 2 consecutive nights out of 1 J.-dlV You and a friend spot a loose $20 bill simultaneously and agree
*eet<end] permission lor each night is obtained (or denied) by to an auction in which you write down your bids and compare
asking either Father or Mother. Father is known to be more likely them. High bidder gets the $20 and pays the other the arnount of
asked on 2
i, giirt permission. However, if the same parent2 isdays the higher bid. Tie bidders split the $20. How much do you bid?
cori"crtiv" days the answers are never the same running'
-Contributed
wh;; should ie ask first? - Contributed

Answer: 9.99 dollars


Answer: Mother

82 B3
I
I
i

ir
li
ii
i

CHAPTER 2 CHAPTER 2
i

27
tli!

D6
i

A list contains 1968 statements, numbered in serial order' For With only a 7 minute and an l l minute ..hour glass', to keep time,
statement is: "This list contains exactly k false you wish to hard boil some eggs for 15 minutes ..on
each k, the kth the nose-,,
statements." Determine the truth or falsity of each statement!
You could start both timers, tt e eggs on when the 7 ninute
frit
timer runs out, invert the other timer at T:l l, and the eggs will
-Contributed be ready when it runs out at T:22. But can the job bi- done
faster?
-Contibuted
Answer: Statement 1967 is true and the rest
are false.

t
I

B4 B5
J*
",ff&d-
CHAPTER 2

2A
['OB GAMBLETS
Problerns inaolaing
p?'obability amd
deteytnim,utiom of the
HardKnoxCollegeisamemberofasix-schoolbasketballleague
twice' The other
rte,Jtnber of outootnes
i" *t rctt "u"ry pii. of schools plays each otherleague
five scnoots end^ed the season *ith respective
.iOo, .:oo, .500, .600, and -800' How did Hard Knox
records of
make out?
of aarious situations
-lontributed

Answer: Tied with second place with 0.600

B6 87
CHAPTER 3

A gambler devised a game to be played with a friend. He bet Vz


the money in his pocket on the toss of a coin; heads he won,
tails he lost. The coin was tossed and the money handed ovrr.
The offer was repeated and the game continued. ,Each tirne the
bet was for Vz the money then in his possession. Eventually the
number of times he lost was equal to the number of tirnes he
won. Quickly now! Did he gain, lose, or break even?
Electronics Monthly
-Pacific

Answer: He lost

I
B9
.@ga.-
CHAPTER 3
CHAPTER 3

l"l
II
,l

Smith and Jones, both 507o marksmen, decide to fight a duel


Aprisonerisgivenlowhiteballs,l0blackballsandtwoboxes. in which they exchange alternate shots until one is hit. What are
fte is totO thai an executioner will draw one ball from one of the the odds in favor of the man who shoots first?* -Contributed
i*o Uo*"t. If it is white, the prisoner will go free; if it is black'
he will die. How should the prisoner arrange the balls in the boxes
to give himself the best chance for survival?
-Georgia Tech Engineer
Answer: 2/3

Answer: 73.7 %

90 9l
CHAPTER 3 CHAPTER 3

There are n points on a circle. A straight line segment is drawn On a certain day, our parking lot contains 999 cars, no two of
between each pair of points. How many intersections are there which have the same'3 digit license number. After 5 : 00 p.m . what
within the circle if no 3 lines are collinear? * -Contributed is the probability that the license numbers of the first 4 cars
to leave the parking lot are in increasing order of magnitude?'l'
- Contributed

Answer: 1 in 24

92 93
CHAPTER 3
CHAPTER 3

A hospital nursery contains only two baby boys; the girls have
Three marksmen simultaneously shoot at and hit a rapidly spinning
not yet been counted. At 2:00 p.m. a new baby is added to the
spherical target. What is the probability that the three points of
nursery. A baby is then selected at random to be the first to have
impact are on the same hemisPhere? + -Contributed its footprint taken. It turns out to be a boy. What is the probability
that the Iast addition to the nursery was a girl? ,,.-Contributed

Answer: Probability is one

Answer : 2/5
tie*Vf*1&

I 95
94
-d***
CHAPTER 3 CHAPTER 3

Assume that a single depth charge has a probability of 1/? of What is the most tikely distribution of the suits in a hand at
sinking a submarine, l/4 of ,Jamage and l/4 of missing. Assume Bridge? (It is not 4-3-3-3.) -Contributed
also that two damaging explosions sink the sub. What is the
probatrility that 4 depth charges will sink the sub? -Contributed
Answer: 715 ways
Answer: Probability of escape - 5/256
Probability of sinking - 251/256

96 97
-d,,1&*%*
tt
CHAPTER 3 CHAPTER 3

to

Using a "true" coin, a random sequence of binary digits can be If 2 marbles are removed at random from a bag containing black
generated by letting, say heads denote zero and tails, one. An oper- ancl white marbles, the chance that they are both whiteis L/3'7f'
ations analyst wished to obtain such a sequence, but he had only 3 are removed at random, the chance that they all are white is l/6'
one coin which he suspected was not true. Could he still do it? How many marbles are there of each color? -Contribuled
-Contributed

Answer: White - 6; Black - 4

98 99
du**
CHAPTER 3 CHAPTER 3

t2 t8

Rigorously speaking, two men are "brothers-in-law" if one is An expert gives team A only a 4O% chance to win the World
married to the full sister of the other. How many men can there Series. Basing his calculation on this a gambler offers 6 to 5 odds
be with each man a brother-in-law of every other man? on team B to win the first game. Is his judgment sound?
* Contributed - Contributed

100 IOl
CHAPTER 3 CHAPTER 3

t1 t6
--li-;'a.--;*
'.
-' i-i (-.;

A salesman visits ten cities arranged in the form of a circle, A coin is so unbalanced that you are likely to get two heads

spending a day in each. He proceeds clockwise from one city to


in two successive throws as you are to get tails in one. What
is the probability of getting heads in a single tbrow?-Contributed
the next, except whenever leaving the tenth city he may go to
either the first or jump to the second city- How many days must
elapse before his location is completely indeterminate, i.e., when
he could be in any one of the ten cities? - Contributed
Answer: 0.618
Answer : 83 days

103
102
CHAPTER 3 CHAPTER 3

t6 t7

Three dart players threw simultaneously at a tic-tac-toe board, Four swimming pool builders submit sealed bids to a homeowner
each hitting a different square. What is the probability that the who is required by law to accept the last bid that he sees, i.e.,
three hits constituted a win at tic-tac-toe? once he looks at a bid, he automatically rejects all previous bids.
-Contributed He is not required to open all the envelopes, of course. Assuming
that all four bids are different, what procedure will maxirnize his
chances of accepting the lowest bid. and what will be the prob-
ability of doing so? - Con trihuted
Answer: 2/21

104 l05
F***."
L
I

CHAPTER 3 CHAPTER 3

IE t9

basketball while al1 but one


All the members of a fraternity play (5
A game of super-dominoes is played with pieces divided into three
ptuy i." hockey; yet the number of possible basketball teams cells instead of the usual two, containing all combinations from
is the same as the number of possible ice hockey teams triple blank to triple six, with no duplications. For example the
-"..ru"rr) set does not include both I 2 3 and 3 2 I since these are merely
(i me.rrU"rr). Assuming there are enough members tb form either
iype of team, how many are in the fraternity? -Con*ibuted reversals of each other. (But, it does contain 1. 3 2.) How many
piec'es are there in a set? -Contributed
Answer: 15 members; 3,003 teams

Answer: 196 pieces in a set

106 t07
I

CHAPTER 3
CHAPTER 3

20 DT

There are three families, each with two sons and two daughters'
Martian coins are 3-sided (heads, tails, and torsos), each side In how many ways can all these young people be married?
coming up with equal probability' Three Martians decided to
go oOJ--utt-out to determine who Pays a dinner check' (If two
-Contributed
Ioins come up the same and one different, the owner of the
latter coin foois the bill). what is the expected number of throws
needed in order to determine a loser? - Contributed Answer: 80

Answer: 1 1/2

109
loB
--Fj-
'l
I

I
,
I
i

CHAPTER 3
CHAPTER 3

qD,qD, ,DB
__

ri
i,1,,

ilH
rl
itH
JE
l[r
iiff
Six men decide to play Russian roulette with a six gun loaded
rlH
llow many three digit telephone area codes are possible given with one cartridge. They draw for position, and afterwards' the
that: (a) ih" fittt digit must not be zeto or one; (b) the second
riE

sixth man casually suggests that instead of letting the chamber


ifir
digit must be zero or one; (c) the third digit must not be zero; rotate in sequence, each man spin the chamber before shooting'
(d) ttre third digit may be one only if the second :;:;:r:,r:;:", How would this improve his chances? - Contrihuted

Answer: Improve by 0.1


Answer: 136 possible codes

I
I

r10
I
I llr
-S@ee.
I
t
I

CHAPTER 3
CHAPTER 3

21 25

.--.,
:-a./'- I
aFl

In Puevigi, the game of craps is played with a referee calling the


A long shot poker player draws two cards to the five and six of point by adding together the six faces (three on each die) visible
diamonds and the joker. What are his chances of coming up with from his vantage point. What is the probability of making 16 the
a pat hand? (straight or flush). *Contributed
-Contrihuted hard way? (That is, by throwing two eights')

Answer: 0.168 Answer: zero

i
I rl3
l12 I

#EE-*
CHAPTER 3 CHAPTER 3

26 27
Ro
h,

Max and his wife Min each toss a pair of dice to determine where There are four volumes of an encyclopedia on a shelf, each
they will spend their vacation. If either of Min's dice displays the volume containing 300 pages, (that is, numbered 1 to 600), but
same number of spots as either of Max's, she wins and they go to these have been placed on the shelf in random order. A book-
Bermuda. Otherwise, they go to Yellowstone. What is the chance worm starts at the first page of Vol. 1 and eats his way through
they'll see "Old Faithful" this year?
* Contributed
to the last page of Vol. 4. What is the expected number of pages
(excluding covers) he has eaten through? - Contributed
Answer: 0.514
Answer: 500

,t

lr4 i t15
,A*
r
I
{
i
i

CHAPTER 3 CHAPTER 3

PE 2,D

Venusian batfish come in three sexes, which are indistinguishable In a carnival game, 12 white balls and 3 black balls are put in
(except by Venusian batfish). How many live specimens must an opaque bottle, shaken up, and drawn out one at a time. The
our astronauts bring home in order for the odds to favor the player gets 25 cents for each white ball which emerges before the
presence of a "mated triple" with its promise of more little batfish first black ball. If he pays one dollar to play, how much can be
to come? *Contributed he expect to win (or lose) on each game?
-(6n1yifiy7af,

Answer: 5 Answer: A loss of a quarter per game

tl6 tt7
CHAPTER 3 CHAPTEH 3

so BT

In the binary system there are only two positive integers con- Which is the more unlikely event in bridge: the ultimate in dis-
taining no digit more than once, namely 1 and 10. How many tribution (a 13 card suit) or the ultimate in point count?
are there in base ten? -lontributed
-Contributed

Answer: 9 digits; 8667690 number of positive


integers all of whose digits are distinct

118 119
r
I

CHAPTER 3
CHAPTER 3

82 8B

rl

What is the longest legal sequence of bids in a single contract


bridge hand? In the final seconds of the game, your favorite N.B,A. team is
behind 1L7 to 118. Your center attempts a shot and is fouled for
-Contributed
the 2nd time in the last 2 minutes as the buzzer sounds. Three to
make two in the penalty situation. Optimistic? Note: the cenler is
only a 5OVo free.thrower. What are your team's overall chances of
Answer: 316 winning?
-Contributed

Answer: 11/16 or 69%

120 121
T

CHAPTER 3 CHAPTER 3

81 B5

ffi,###w#
One of a pair of dice is loaded so that the chance
of a I turning If all 720 permutations of the digits 1 through 6 are arranged in
up is 1/5, the other faces being equally likely. Its
mate is loaded numerical order, what,is the 417th term?
so that the chance of a 6 turniig up is"1/5, the
other i";;, Gi;; -Contributetl
equally likely. How much. does this loading increase
tt p.otl
ability of throwing a 7 wjth the two rlicei ' "
_Contributed
Answer: 432516

Answer: 1/750

122 123
-:flF+--

CHAPTER 3
CHAPTER 3

a6 87

The local weather forecaster says "no rain" and his record is 2/a
accuracy of prediction. But the Federal Meteorological Service A sharp operator makes the following deal. A player is to toss a
predicts rain and their record is 7+. With no other data availa- coin and receive 1,4,9, .. . n2 dollars if the first head comes up on
ble, what is the chance of rain? the first, second, third, . . . n-th toss. The sucker pays ten dollars
--lontributed for this. How much can the operator expect to make if this is re"
peated a great many times?
-Contributed

Answer: 3/5 Answer: 4 dollars per game

t24 I
rzs
&*
CHAPTER 3 CHAPTER 3

8E 89

In 1969, the World Series began in the stadium of the Amer- In a carnival game 5 balls are tossed into a square box divided
ican League pennant winner. Assume the contenders are evenly into 4 square cells, with baffes to insure that every ball has an
matched. What is the probability that the series ended where equal chance of going in any cell. The player pays $ I and receives
it began? $1 for every cell which is empty after the 5 balts are thrown. How
-Contributed much does the operator expect to make per game?-lontributed

Answer: 5/8
Answer: A profit of a nickel per game on
average ( 5 cents or 0.05 dollars)

t26 127
CHAPTER 3

10
r'ORTHM
AI,GEBRATCALLY
INCI,TlUEI)
i$-E\,,=Y=l- I Probleruts
Having lost a checker game, a specialist in learning programs
threw one of the red checkers out the window. His wife reboxed
best uttaoked, by
?nea,rus of oariables
the 1,2 black pieces and 11 red pieces one at a time in random
fashion. The number of black checkers in the box always ex-
ceeded the number of reds. What was the a priori probability of
this occurrence?
-Contributed
by Les Marvin
arud equatioms
Answer: Desired Probability - 1/23
Bad probability - 22/23

128
129
CHAPTER 4

l;
t:i
tlt

ri

,li
il
,1

To stimulate his son in the pursuit of partial differential equa-


tions, a math professor offered to pay him $8 for every equation
correctly solved and to fine him $5 for every incorrect solution.
At the end of 26 problems, neither owed any money to the other.
How many did the boy solve correctly?
-Adapted lrom Clavias, 1608

Answer: 10 problems

r3t
CHAPTER 4
CHAPTER 4

An expert on transformer design relaxed one Saturday by going A mathematician whose clock had stopped wound it, but did Dot
to the races. At the end of the first race he had doubled his bother to set it correctly. Then he walked from his home to the home
money. He bet $30 on the second race and tripled his money. of a friend for an evening of hi-fi music. Afterwards, he walked back
He bet $54 on the third race and quadrupled his money. He bet to his own home and set his clock exactly. How could he do this
$72 on the fourth race and lost it, but still had $48 left. With without knowing the time his trip took?
how much money did he start?
-adapted lrom the ancient Hindu
from Chuquet, 1484
-Adapted

Answer: 29 dollars

132 133
Ii ]]ilr-
tli
;i'
:l

ii
ll

CHAPTER 4
CHAPTER 4

re :'/'

Between Sing-Sing and Tarry-Town, ForX (1 evaluatetheinfiniteproduct: (l + X +X'+ -" +f)


I met my worthy friend, John Brown, (1 +X*+X'+X' + ... +x-) (1 +x* + x*+. .' +x*)
And seven daughters, riding nags, (. .. {,
- M ath emat bs lul a Sazi ne
And every one had seven bags.
In every bag were thirty cats,
And every cat had fortY rats,
Besides a brood of fifty kittens.
Nl but the nags were wearing mittens!
Mittens, kittens cats, rats bag, nags Browns,
How many were- met between- the towns? * -
67 6e17's C ornmon Arithmetic
-lel[

Answer: 764, 488

I35
134
CHAPTER 4 CHAPTER 4

If v varies as w2, w3 as x4, x5 as y0, and y7 as 24, show that the Dr. Reed, arriving late at the lab one morning, pulled out his
watch and said, "I must have it seen to. I have noticed that the
product I .
1"/
- I . I do". not vary at all.
'zzzz -Contributed minute and the hour hand are exactly together every sixty-five
minutesl' Does.Dr. Reed's watch gain or lose, and how rnuch
per hour? x
-Contibuted

Answer: A gain of 60/143 minutes per hour

t36 137
,,
CHAPTER 4 CHAPTER 4

At this moment, the hands of a clock in the course of normal Two men are walking towards each other at the side of a railway.
operation describe a time somewhere between 4:00 and 5:00 A freight train overtakes one of them in 20 seconds and exactly
on a standard clock face. Within one hour or less, the hands will ten minutes later meets the other man coming in the opposite
have exactly exchanged positions; what time is it now? direction. The train passes this man in l8 seconds. How long after
-Contributed the train has passed the second man will the two men meet?
(Constant speeds are to be assumed throughout.) *
-Contilbuted
Answer: 4:26.8531
Answer: 5562 seconds

138 139
CHAPTER 4
CHAPTER 4

TO TI

Using the French Tricolor a's a model, how many flags are pos- A cubic box with sides 'a' feet long is placed flat against a wall.
sible with five available colors if two adjacent rows must not be A ladder-'p'.feet'long is placed in such a.way that it touches the
colored the same?* -Contributed,
)vall as well' as the free horizontal edge of the box. If a = 1 and
p - VTt calculate at what height the ladder touches the wall,
*
using quadratics only.
-Contributed

Answer: 50 flags
Answer: 3.62 or 1.38 feet

{F.- " ,"{*

i
4.
I
I

140 141
CHAPTER 4 CHAPTEH 4

I2 I8

Dr. Irving Weiman, who is always in a hurry, walks up an up- Citizens of Franistan pay as much income tax (percentage-wise)
going escalator at the rate of one step per second. Twenty steps as they make rupees per week. What is the optimal salary in
bring him to the top. Next day he goes up at two steps per second, Franistan? x
reaching the top in 32 steps. How many steps are there in the -Contribated
escalator? Contributed
n..!1 - ',-{-
Answer: 50 rupees
Answer: 80 steps
l'm
ffit l1 ,..1

t"56

l$*u$*tl

142 143
CHAPTER 4 CHAPTER 4

T1 l5

There are nine cities which are served by two competing airlines. Two snails start from the same point in opposite directions toward
One or the other airline (but not both) has a flight between every two bits of food. Each reaches his destination in one hour. If each
pair of cities. What is the minimum number of triangular flights snail had gone in the direction the other took, the first snail would
(i.e., trips from A to B to C and back to A on the same airline)? * have reached his food 35 minutes after the second. How do their
speeds compare? Contributed
-Contributed -

Answer: 12, number of triangular flights


Answer: The first snail traveled 3/4 the speed of
the second snail

t44 t45
I
{
{

CHAPTER 4 CHAPTER 4

I6 l7

iltt,t

A of pearls which increase uniformly from a


necklace consists A pupil wrote on the blackboard a series of fractions having
weight of for the end pearls to a weight of 100 carats
1 carat positive integral terms and connected by signs which were either
for the middle pearl. If the necklace weighs altogether 1650 all * or all X, although they were so carelessly written it was
carats and the clasp and string together weigh as much (in carats) impossible to tell which they were. It still wasn't clear even
as the total number of pearls, how many pearls does the necklace though he announced the result of the operation at every step.
contain? - Contributed The third fraction had denominator 19. What was the numerator?
-Conlributed

Answer: 33 pearls, an increment of 99/16 carats Answer: 25, First fraction was 5/3, Second fraction
was 5/2 and third fraction was 25/19

t46 L47
CHAPTER 4

t9
CHAPTER 4

IE

all' Archimedes
Without performing any algebraic manipulation.at
Jai Alai balls come in boxes of 8 and 15; so that 38 balls
(one
that the sum and product of the two expresslons
O'toole remarked
small box and two large) can be bought without having to break
openabox,butnot3g.Whatisthemaximumnumberofballs
x+ v--l x -v I t-LlEj are respectivelyxi-y and
be bought without breaking boxes? -Contributed
""d
*hi"h xy. Why was this obvious? -Contributed
"uorrot

Answer: 97

149
r48
CHAPTER 4 CHAPTER 4

20 2I

A parking lot charges X for the first hour or fraction of an hour Mr. Field, a speeder, travels on a busy highway having the same
and,TtX for each hour or fraction thereafter. Smith parks 7 times rate of traffic flow in each direction. Except for Mr. Field, the
as long as Jones, but pays only 3 times as much. How long did traffic is moving at the legal speed limit. Mr. Field passes one
each park? (The time clock registers only in 5-minute intervals.) car for every nine which he meets from the opposite direction.
By what percentage is he exceeding the speed limit?
-C<tntributed
-Contributed
Answer: Jones - Half an hour (30
minutes); Smith - 3.5 hours
Answer: 25%

i
l

i
i
i
1

1
iI
I
l
k 151
r50 ,&,.
','
CHAPTER 4

p8
CHAPTER 4

qD,qD,
--,

i?,2',?,3'fr#r
The teacher marked the quiz on the following basis: one point
What is the millionth term of the sequence l, 2, 2,3, 3, 3, 4, 4,
4, 4,...in which each positive integer n occurs in blocks of n for each correct answer, one point off for each question left
blank and two points off for each question answered incorrectly.
terms? - Contribtded Pat made four times as many errors as Mike, but Mike left nine
more questions blank. If they both got the same score, how many
errors did each make? -Contributed
Answer: 1,414
Answer: Pat - 8 errors, Mike - 2 errors

153
ts2
#***.
CHAPTER 4

p5
CHAPTER 4

D1

ftm*ffiem#hm
A student, just beginning the study of logarithms, was required to There are four towns at the corners of a square. Four motorists
evaluate an expression of the form '1ff*. ,. proceeded to cancel set out, each driving to the next (clockwise) town, and each rnan
common factors in both numerator and denominator, (including but the fourth going 8 mi./hr. faster than the car ahead-thus the
the "factor" log), and arrived at the result ]. Surprisingly, this first car travels 24 mi./hr. faster than the fourth. At the end of
one hour the first and third cars are 2O4, and the second and fourth
was correct. What were the values of A and B? -Contributed
212 (beeline) miles apart. How fast is the first car traveling and
how far apart are the towns? -Contributed

Answer: A = 9/4, B = 27/8


Answer: 50 mph, distance between towns = 180
miles

154 r55
r
I

CHAPTER 4
CHAPTER 4

26 27
2 7, '6
I'4 _5, I
,3 :8

Represented above is a "magic square" in which the sum of each


row, column, or main diagonal is the same. Using nine different Solve the equat"ion \/;+J; I vV+: : J - --Cinnibutert
VT: where
integers, produce a "multiplicative" magic square, i.e., one in both members represent infinite expressions. "r/*
which the word "product"is substituted for"sum".-Contributed

Answer: x = 0 or 2

156
L57
CHAPTER 4

pg
CHAPTER

2t
4

UflMUflM,UflM,Uffi
3+...
4
3 *--------;-
4
3*-----,-
4 A pencil, eraser and notebook together cost $1.00. A notebook
-3 r 3.r- costs more than two pencils, and three pencils cost more ihan four
erasers. If three erasers cost more than a notebook, how much does
- and numerators continue in-
where the respective denominators
each cost?
definitely? Contributed -Contributed
-
Answer: Pencil = 26, Eraser = 19, Notebook
Answer: Equal = 55

I.58
A 159
GHAPTER 4 CHAPTEFI 4

ao 8I

Depicted above are two interlocked hyperbolas. Impossible?' Solve for real values of x: (7 * a f-3x - 4(Z + f,f: lx = -1.
You're right, but can you prove it? by Stephen A. Cooperman
-Contributed -Contributed

Answer: x = 1

160 161
CHAPTER 4

B2
f.'OBTHE
GBOITTETRICALLf
INCLTNEI)

Problerns im,aolainog
In Puevigi numbers such as 2, 5, 8, 10, etc., that are the sum of
two squares, are considered sacred. Prove that the product of
plam,e geonoett'A aTrd
any number of sacred numbers is sacred.
-
()ontribuled bv Charles C" Foster trigorr,otttetry

I62 i 163
J.
I
m*ik_

CHAPTER 5

A wizard in Numerical Analysis has a gold chain with 7 links' A Lady


programmer challenges him to use the chain to buy 7 kisses, each kiss
to be paid for, separately, with one chain link. what is the smallest
number of cuts he will have to make in the chain? what is his se-
quence of payments ? - adapted lrom the U niversitv ol Calilornia Engineer

I
165
t_
TFp--

CHAPTER 5 CHAPTEH 5

A forgetful physicist forgot his watch one day and asked an E.E. The faces of a solid figure are all triangles. The figure has nine
on the staff what time it was. The E.E. Iooked at his watch and vertices. At each of six of these vertices, four faces meet, and at
said: "The hour, minute, and sweep second hands are as close to each of the other three vertices, six faces meet. How many faces
trisecting the face as they ever come. This happens only twice in does the figure have? x
-Contributed
every 12 hours, but since you probably haven't forgotten whether
you ate lunch, you should be able to calculate the timel' What
time was it to the nearest second? _Contributed Answer: 14 faces, 21 edges

Answer: 2 hours 54 minutes 35 seconds


or 9 hours 5 minutes 25 seconds

r66 167
,=
$lti
I itr
Iif,;
liill

CHAPTER 5 CHAPTER 5 $ffi


il[

{rI

$i{
fitt

Hit
utl
li l

----'' lill
fll
fi11

ffi]l
fllI

ffi!1

htq

hl11
rl,l
ffil11
A new kind of atom smasher is to be composed of two tangents A spider and a fly are located at opposite vertices of a room of rlil11

and a circular arc which is concave towards the point of inter- dimensions 1,2 and 3 units. Assuming thet the fly is tooterrified
section of the two tangents. Each tangent and the arc of the circle to move, find the minimum distance the spider must crawl to reach [li
is I mile long. What is the radius of the circle?* the fly. -Contributed
-A me ri ca n M at he mat ical M onth ly i,li

n,i

Answer: 1437.45 feet Answer: Square root of 18 units 1i

hl*u t* r; *"

168 t69
'@*.
CHAPTER 5 CHAPTER 5

Show that tan is a root of the equation 5xa l0x2 f I: * a bug sits
+ -
*
0.
Contributed
In a room 40 feet long, 20 feet wide, and 20 feet high,
on an end wall at a point one foot from the floor, midway between
the sidewalls. He decides to go on a journey to a point on the other
end wall which is one foot from the ceiling midway between the
sidewalls. Having no wings, the bug must make this trip by
sticking to the surfaces of the room. What is the shortest route
that the bug can take? - Contrihuted

Answer: 58 feet

gBffiwp* tl
t.,\'**- \ra*$l'

170 t7] \. Iiitl..::r


i .:{i}-x:
CHAPTER 5
CHAPTER 5

A circle of radius I inch is inscribed in an equilateral triangle. A A farmer owned a square field measuring exactly 2261 yards on
smaller circle is inscribed at each vertex, tangent to the circle and each side. 1898 yards from one corner and 1009 yards from an
two sides of the triangle. The process is continued with progress_ adjacent corner stood a beech tree. A neighbor offered to pur-
ively smaller circles. what is the sum of the circumference of all chase a triangular portion of the field stipulating that a fence
circles? Contributed
should be erected in a straight line from one side of the feld to
an adjacent side so that the beech tree was part of the fence. The
farmer accepted the offer but made sure that the triangular por-
Answer: 5 pi incehs tion was of minimum area. What was the area of the field the
neighbor received, and how long was the fence? _Contributed

Answer: Area - 939, 120 square yards


Length of fence - 2018 yards
tt*
m'' * \6;,r.

172 173
I

I
I

rt
CHAPTER 5 CHAPTER 5

to

Given five points in or on a unt t square, prove that at least two Given a point P on one side of a general triangle ABC, construct
points are no farther than \/T units apart. * -Contrihuted a line through P which will divide the area of the triangle into
-2 two equal halves. x
-Coatrihuted

174 t
I
t75
t.
,-}.3htu
I

CHAPTER 5 CHAPTER 5

IP t8

llillilll
A man leaves from the point where the prime meridian crosses Near the town of Lunch, Nebraska there is a large triangular
the equator and moves forty-five degrees northeast by geographic plot of land bounded by three straight roads which are 855,
compass which always points toward the north geographic pole. 870, and 975 yards long respectively. The owner of the land,
He constantly corrects his route. Assuming that he walks with a friend of mine, told me that he had decided to sell half the plot
equal facility on land and sea, where does he end up and how to a neighbor, but that the buyer had stipulated that the seller
far will he have travelled when he gets there? -Cotntributed of the lancl should erect the fence which was to be a straight one'
The cost of fences being high, my friend naturally wanted the
fence to be as short as possible. What is the minirnurn length
Answer: North Pole at a distance of square root of the fence can be? -Contributed
2 times 10^7 meters.

Answer: 600 yards

t76 177
CHAPTER 5 CHAPTER 5

I1 t5

Three hares are standing in a triangular field which is exactly


100 yards on each side. One hare stands at each corner; and
A scalene triangle ABC which not a right triangle has sides
\_is.
simultaneously all three set off running. Each hare runs after which are integers. If sin A: fr, nna the smallest values for its
the hare in the adjacent corner on his left, thus following a
sides, i.e., those values which make the perimeter a minimurn'
curved course which terminates in the middle of the field, all * Contibuted
three hares arriving there together. The hares obviously ran
at the same speed, but just how far did they run?
-Contributed
Answer: a = 25, b = 16, c = 39

Answer: Exactly 100 yards

I
I
178 I
I
t79
I
CHAPTER 5
CHAPTER 5

I6 t7

A divided highway goes under a number of bridges, the arch over


A one-acre field in the shape of a right triangle has a post at the
midpoint of each side. A sheep is tethered to each of the side each lane being in the form of a semi-ellipse with the height equal
posts and a goat to the post on the hypotenuse. The ropes are to the width. A truck is 6 ft. wide and 12 ft. high. What is the
just long enough to let each animal reach the two adjacent vertices. lowest bridge under which it can pass? * -Contributed
What is the total area the two sheep have to themselves, i.e", the
area the goat cannot reach? x -Contributed Answer: 6 square root of 5 or 13 feet and 5
inches

Answer: Exactly 1 acre

tBl
rBo
CHAPTER 5 CHAPTER 5

IE t9

A cowboy is five miles south ot a stream which flows due east. While still at a sizable distance from the Pentagon building, a man
He is also 8 miles west and 6 miles north of his cabin. He wishes first catches sight of it. Is he more likely to be able to see lwo sides
to water his horse at the stream and return home. What is the or three?
shortest distance he can travel and accomplish this? -Contibuted
-Contributed

Answer: 8 square root of 5 miles or 17.9


miles.

t82 I83
&
-""&.,
CHAPTER 5 CHAPTER 5

20 2t

236
A pirate buried his treasure on an island, a conspicuous landmark An Origami expert started making a Nani-des-ka by folding the
of which were three palm trees, each one 100 feet from the other top leficorn". of u sheet of paper until it touched the right edge
two. Two of these trees were in a N-S ]ine. The directions for u.rO th" crease passed through the bottom left corner' ]Ie then
finding the treasure read: "Proceed from southernmost tree 15 did the same with the lower right corner, thus making tvo slant-
feet due north, then 26 feet due west." Is the treasure buried ing parallel lines. The paper was 25 inches long and the ilistance
within the triangle formed by the trees? - Contribute.d between the parallel lines was exactly of the width' How wide
fr
was the sheet of paper? -Cortributeil

Answer: The treasure will lie outside the triangle they


form Answer: 24 inches

lB4 185
."L
CHAPTER 5 CHAPTER 5

ED,qD,
--
28

ww 248
$"
$,\
250
The Ben Azouli are camped at an oasis 45 miles west of Taqaba. A cross section through'the center of a football is a circlex inches
They decide to dynamite the Trans-Hadramaut railroad joining in circumference. The football is x-8 inches long from tip to tip
Taqaba to Maqaba, 60 miles north of the oasis. If the Azouli can
I

and each seam is an arc of a circle x inches in diameter' Find x'


cover 18 miles a day,,how long wlll it take them to reach the f,
railroad? -Contfibuted
r\ - Contributed
! i..t Answer: x = 20.69 inches

Answer: D = 36 miles, 2 days

#:
}{&uffi 1 l
i

186 r87
I
J.
CHAPTER 5 CHAPTER 5

21 25

282
284
Let c be the hypotenuse of a right triangle with legs a and b. A yang, ying, and yung is constructed by dividing a diameter
Prove that if x) 2, then ax * b* 4 g*.
Contributed of a circle, AB, into three parts by points C and D, then describ-
-
ing on one side of AB semicircles having AC and AD as diameters
and on the other side of AB semicircles having BD and BC as
diameters. Which is larger, the central portion or one of the
outside pieces? - Conlributed

Answer: All three are the same size equal to 1/3pi R^2

IBB I

r89
A
CHAPTER 5

p6
CHAPTEH 5

27

307 316
A diaper is in the shapeof a triangle with sides 24, Z0 and 20 A coffee pot with a circular bottom tapers uniformly to a circular
inches. The long side is wrapped around the baby,s waist and top with radius half that of the base. A mark halfway up the side
overlapped two inches. The third point is brought up to the cen- says "2 cups." Where should the "3 cups" mark go? Conrribated
ter of the overlap and pinned in place. The pin is to go through
three thicknesses of material. What is the area in which the pin
may be placed? Answer: 2% way down from the top
-'Contributed
Capacity of the cup = 3 1/37 cups

Answer: 2 1/2 square inches

190
i
I
I
r9l
"t
,fu"
CHAPTER 5 CHAPTER 5

2E Dg

lr' r

318
183
How can seven points be placed, no three on the same line, so An icicle forming from a dripping gutter is in the shape of a cone
that every selection of three points constitutes the vertices of an five times as long as it is wide (at the top). A few hours later it
isoceles triangle? has doubled in length and the generating angle has also doubled.
-Contributed
How does its present weight compare with its previous weight?
-Contributed
Answer: Almost 33 times its previous weight

192 I r93
&
CHAPTER 5 CHAPTER 5

BO 8T

7,4-

332 356
A hostess plans to serve a square cake with icing on top and sides. Prove that each median of a triangle is shorter than the lverage of
Upon determining how many guests want cake, what method the 2 adjacent sides"
should she use to insure that each guest will receive the same
amount of cake and icing?
-Contributed

194 r95
CHAPTER 5
CHAPTEB 5

B2 BB

420 441
Define every point of the plane with 2 integer coordinates (e.g.
[3,0] or l-5,21) as a "lattice point." Let every pair of lattice
A student beginning the study of trigonometry came aDross an
points in the plane be connected with a "lattice line." Prove or expression of the form sin (Xl-Y). He evaluated this as sin
disprove: "The Iattice lines cover the plane." X*sin Surprisingly he was correct. The values of f, and Y
Y.
-Contributed differed by l0'; what were these values, assuming that D"<X<
Y<360"?
-Catjtributed

Answer: 175 and 185

196 197
CHAPTER 5
CHAPTER 5

81 B5

dx
454 462
Above is a map of Lake Puevigi. The cross represents a buried If the equal sides of an isosceles triangle are given, what length
treasure cache" Cover the right hand half of the diagram. Now of the third side will provide maximum area? (No calculus,
answer: "Is the treasure in the lake or on land?" -lontributed please. )
-Contributed

Answer: Square root of 2 times the other


two legs

t
--'_*_" "'t'_1
'""''"'|' o,ro. j
;"C$dfl
rj d
i .*,, -*'"'"

I98 i 199
I
J*
-r'rfr-

CHAPTER 5 CHAPTER 5

86 B7

470 481
One side of a triangle is l0 feet longer than another and the angle Here's a rather unusual optical illusion. How many different con-
between them is 60". Two circles are drawn with.these sides as flgurations can you "see"?
diameters. One of the points of intersection of the two circles is -Contfibuted
the common vertex. How far from the third side is the other point
of intersection?
-Contributed

Answer: Distance is equal to zero. Point of


intersection will be on the third side.

200 201
,#aj*,
T
CHAPTER 5
CHAPTER 5

AE Bg ti
ii

498
499
The isosceles right triangle shown above has a vertex at the cen-
ter of the square. What is the area of the common quadrilateral? There is one flag at the entrance to a racetrack and another in-
side the track, half a mile from the first. A jockey notes that no
-Contributed matter where he is on the track, one flag is 3 times as far away
as the other. How long is the track?
-Contributed
Answer: 12.25 square units

Answer: 1980 pi ft or 3.36 furlongs

2o.2
203
"rr*&*,
-1"-
i

CHAPTER 5 CHAPTER 5

10 1I

504
Through binoculars a bird watcher observed a hummingbird feeder What is the longest 6' wide shuffie board court which will fit in
between one and two o'clock of an afternoon. He timed the visits a20'x 30' rectangular room? __{ontributed by Carl Sorber
and saw a ruby-throat take a drink at 1,5,6,8, 15, 16, 19,22,
27, 29, 32, 36, 38, 43, 45, 49, 50, 57, and 58 minutes after the
hour of one. The last visit he saw took place at two, at which time
he left in perplexity. He knew from experience that a hummer's Answer: 30 7/8 feet
"feeding cycle" is remarkably stable and is generally between 5
and 15 minutes long. This one seemed rather erratic, to say the
least. Can you advise him on what was going on?-Contributed

204 205
,-tly#es-
B'OB f}TiTS
Of'DTOPIIANTUS

Probletns i,m,aolaing
I)iophamtim,e
eqltoatiorts, those for
trshieh omly whole
miurnhey soltttiotts
ure soeoght

207
CHAPTER 6

Find the smallest number (x) of persons a boat may carry so that (n )
married couples may cross a river ip such a way that no woman ever
remains in the company of any man unless her husband is present.
Also find the least number of passages (y) needed from one bank to
the other. Assume that the boat can be rowed by one person only.
- E. Lucas, 1883

209
_-r--
I

I
I

CHAPTER 6
CHAPTER 6

il

ir
: _:\\.
\+,

57
112
A, B, and C participate in a track meet, consisting of at least
three events. A certain number of points are given for first Find the simplest solution in integers for the equation
place, a smaller number for second place, and a still smaller ! _ll _1.+.
number for third place. A won the meet with a total score of
x2 'Y2-22 -Contributed

14 points; B and C are tied for second with 7 points apiece. B


won first place in the high jump. Who won the pole vault as-
suming no ties occurred in any event?
-Contributed Answer: x = 15, y = 20, z = 12

Answer: A won the pole vault

I
i

zto I
t
o"kru*l
2tt
a"gr'
CHAPTER 6 CHAPTER 6

124 129
Maynard the Census Taker visited a house and was told,."Three Prove that the product of 4 consecutive positive integers cannot
people live there. The product of their ages is 1296, and the sum be a perfect square. x
-Contilbuled
of their ages is our house numberl'After an hour.of cogitation
Maynard returned for more information. The house owner
said, "I forgot to tell you that my son and grandson live here with
mel' How old were the occupants and what was their street
n umber ?
- Contrihuted

Answer: Ages are 1, 8, and 72, House


number = 91

I
|'

I
I
F

2t2 I
t.
213
ffi,
. ,*tr --{qb".
CHAPTER 6 CHAPTER 6

201
167
In Byzantine basketball there are 35 scores which are impossible Gherkin Gesundheit, a brilliant graduate mathematics student,
for a team to total, one of them being 58. Naturally a free throw was working on an assignment but, being a bit absent.minded,
is worth fewer points than a field goal. What is the point value he forgot whether he was to add or to multiply the three different
of each? *
- Contributed integers on his paper" He decided to do it both ways and, rnuch
to his surprise, the answer was the same. What were the three
different integers? + Cortributed
-
Answer: Free throw = 8, Field goal = 11

Answer: 1, 2, and 3

2L4 2ts
CHAPTER 6 CHAPTER 6

181
Three farmers, Adams, Brown and Clark all have farms contain- 1960 and 1961 were bad years for ice cream sales but 1962 was
ing the same number of acres. Adams' farm is most nearly square, very good. An accountant was looking at the tonnage sold in
the length being only 8 miles longer than the width. Clark has the each year and noticed that the digital sum of the tonnage sold in
most oblong farm, the length being 34 miles longer than the width. 1962 was three times as much as the digital sum of the tcnnage
Brown's farm is intermediate between these two, the length being sold in 1961. Moreover, if the amount sold in 1960 (346 tons),
28 miles longer than the width. If all the dimensions are in exact was added to the 1961 tonnage, this total was less than the total
miles, what is the size of each farm? - Contributed tonnage sold in 1962 by the digital sum of the tonnage sold in
that same year. Just how many more tons of ice cream were sold
in 1962 than in the previous year? * Contributed
-
Answer: A = 40 x 48; B = 32 x 60; C = 30 x 64
Answer: 361 tons

216 2t7
CHAPTER 6 CHAPTER 6

to IT

,,,;li
215 232
Three rectangles of integer sides have identical areas. The first When little Willie had sold all his lemonade he found he had
rectangle is 278 feet longer than wide. The second rectangle is $7.95 in nickels, dimes and quarters. There were 47 coins alto-
96 feet longer than wide. The third rectangle is 542 feet longer gether and, having just started to study geometry, he noticed
than wide. Find the area and dimensions of the rectangles. that the numbers of coins satisfied a triangle inequality, i.e., the
-Contributed sum of any two denominations was greater than the third. IIow
many of each were there?
-Contributed

Answer: Area = 1, 466, 640 square feet


Rectangle 1 = 1080 by 1358 Answer: D = 20; Q = 23; N = 4
Rectangle 2 = 1164 by 1260
Rectangle 3 = 970 by 1512

2lB 2r9
_=-
I

CHAPTER 6 CHAPTER 6

t2 TB

275
266
There are 100 coins in a piggy bank totaling $5.00 in value, the Every year an engineering consultant pays a bonus of $300 to
coins consisting of pennies, dimes and half dollars. How many hls most induslrious assistant, and $75 each to the rest of his
of each are there? -Contributed flutr. aft". how many years would his outlay be exactly $6,000
tt all but two of his staff had merited the $300 bonus, but none
of them more than twice?
-Contributed
Answer: 1 half dollar; 39 dimes, 60 pennies

Answer: n = 8, x = 7 where n is the number of years and x is the


number of staffs

224 22L
CHAPTER 6 CHAPTER 6

T1 l5

303 317
In Eurepean countries the decimal point is often written a little A certain 3-digit number in base 10 with no repeated digits can
above the line.An American, seeing a number written this way, be expressed in base R by reversing the digits. Find the smallest
with one digit on each side of the decimal point, assumed the value of R. _Contfibured
numbers were to be multiplied. He obtained a two-digit number
as a result, but was 14.6 off. What was the original number?
- Contributed
Answer: R = 438 to the base 14

Answer: 5.4

,,, 223
I

CHAPTER 6 CHAPTER 6

I6 t7

342 425
Two wheels in the same plane are mounted on shafts 13 in. apart. Five points are located in or on the perimeter of an equilateral
A belt goes around both wheels to transmit power from one to the triangle with f-inch sides. If d is the distance between the closest
other. The radii of the two wheels and the length of the belt not pair of points, what is the maximum possible value of d?
in contact with the wheels at any moment are all integers- How
much larger is one wheel than the other?
-Contilbuted
-Contributed
Answer: 4.5 inches

Answer: 5 inches larger than the other

I
I

I
I
I
224 I 225
m,
tt
CHAPTER 6 GHAPTER 6

T9

'A
483
If THAT:(AH) (HA), whatisTHAT? A group of hippies are pondering whether to move to Patria,
-Contributed where polygamy is practiced but polyandry and spinsterhood are
prohibited, or Matria, where polyandry is permitted and polyg-
amy and bachelorhood are proscribed. In either event the pos-
sible number of "arrangementsi' is the same. The girls outnurnber
the boys. How many are there? --4ontributed

jl

lr

I
Answer: 4 girls and 2 boys
l

ti

226 i
I 227
&*
CHAPTER 6

20
H'OBNTUMBEB
THEORTSTS

513
ffi
Dad and his son have the same birthday. On the last one, Dad
Pt'obletns
im,aolaing pyoperties
was twice as old as Junior. Ijncle observed that this was the
ninth occasion on which Dad's birthday age had been an integer
of integet a
multiple of Junior's. How old iq Junior?
by R. S. Fieltl,lr.
-Contrihuted

Answer: Junior = 36 yrs old; Dad = 72 yrs old

224 Ito
CHAPTER 7

The undergraduates of a School. of Engineering wished to form ranks


for a parade. In ranks of 3 abreast, 2 men were left over; jn ranks of
5,4 over; in 7's,6 over; and 1l's, l0 over. What is the least number
of marchers there must have been ?
-The Georgia Tcch Engineer

Answer: 1154

231
A
l
CHAPTER 7 CHAPTER 7

18
What is the remainder upon dividing Seee,eee by 7'l A pet store offered a baby monkey for sale at $1.25. The rnonkey
-Fermat, circa 1635
grew. Next week it was offered at $1.89, then $5.13, then $5.94,
then $9.18 and on thesixth week a Ph.D. in Aeronautics bought il
for $12.42. How were the new prices figured?
U -C.L.A. Engineering Student Newslelter
Answer: 6

232 2.33
CHAPTER 7 CHAPTER 7

The odd digits l, 3, 5, 7, and 9, add up to 25, while the even Assume the universe is a billion billion light years in iliarneter
figures, 2, 4, 6, and 8, only add up 20. Arrange these figures so
and is packed solidly with matter weighing a billion billion tons
that the odd ones and the even ones add up alike. Complex and per cubic inch and each gram of this matter contains a billion
improper fractions and recurring decimals are not allowed.
billion atoms. Also, every second during the past billion billion
-Contributed, years, a billion billion similar universes were created- Without
using any symbols and restricting yourself to a total of three
digits, write a number that far exceeds the total atoms of all
these universes. *Contributed

i
I
t
t

!."

234 4.JJ
CHAPTER 7 CHAPTER 7

,ii

$ll

fl
The sum of the digits on the odometer in my car (which reads How many primes are in the following infinite series where the
Ii up to 99999.9 miles) has never been higher than it is now, but it digits are arranged in declining order? 9; 98; 987;98761 --.-------...-;
ll
was the same 900 miles ago. How many miles must I drive before
987654321; 9876543219; 98165432198;. . . etc. * -Contributed
li
it is higher than it is now. -Contributed

Answer: No prime numbers


Answer: 100 miles

.l !l I

236 237
CHAPTER 7 CHAPTER 7

What is the largest number which can be obtained as the product The first expedition to Mars found only the ruins of a civilization.
of positive integers which add up to 100? x
- Contributed The explorers were able to translate a Martian equation as follows:
5x' - 50x * 125 : o x = . fnir was strange mathematics. The
{!
value x : 5 seemed legitimate enough but x: 8 required some
explanation. If the Martian number system developed in amanner
Answer: 3^32 x 2^2 similar to ours, how many fingers would you say the Martians
had? x
-THE BENT of Tar Beta Pi

Answer: 13 fingers

238 239
CHAPTER 7 CHAPTER 7

ro IT

A rectangular picture, each of whose dimensions is an integral Find unequal rational numbers, a, b, (other than 2 and 4) such
number of inches, has an ordinary rectangular frame I inch wide. that ab = b'. x - Contributed
Find the dimensions of the picture if the area of the picture and
the area of the frame are equal. * Contributetl
-

.t Answer: a = 9/4 and b = 27/8


i
Answer: 3 by 10 or 4 by 6 I
I
t
I
,

!
l

i
li t-
i. ,"-

Li^, - 4{ *"5 -' ti L" 'r xT 's'r "'3'-4"9


241
CHAPTER 7 CHAPTER 7

IP I8
7-'-

I ,.,1[ ],,.
Find a five digit number whose first two digits, central digit, and My house is on a road where the numbers run 1, 2,3,4.,-con-
last two digits are perfect squares and whose square root is a secutively. My number is a three digit one and, by a curious coin-
prime palindrome. Contributed
- cidence, the sum of all house numbers less than mine is the same
as the sum of all house numbers greater than mine. What is my
number and how many houses are there on my road?*
-Contributed
Answer: 36481
Answer: House number is 204 and there are
288 houses

242 243
CHAPTER 7 CHAPTER 7

t1 t5

The sum and difference of two squares may be primes : 4 I : 3 On what days of the week can the first day of a century fall?
-
and 4-l-. I : 5; 9 4 - 5 and 9 * 4 : 13, etc. Can the sum and (The first day of the twentieth century was Jan. 1, 1901)r.
-
difference of two primes be squares? If so, for how many differ-
-Contribuled
ent primes is this possible?*
-Contributed
Answer: Monday, Tuesday, Thursday or Saturday
Answer: p = 2 & q = 2 First day of 21st century on Monday

I
I

I
I
!

I
244 h 245
"#14tu-
CHAPTER 7 CHAPTER 7

I6 t7

ffiffi
Solve for A and B, both triangular numbers: 7993 :A2-82. A certain 6-digit number is a square in both the scale of 5 and
-Contributed the scale of 10. What is it?
-Contributed

Answer: A = 319,600 & B = 318,801

246 247
CHAPTER 7 CHAPTER 7

IE t9
/

Starting with one, place each succeeding integer in one of two In a lottery the total prize money available was a milljon dollars,
groups such that neither group contains three integers in arithmetic paid out in prizes which were powers of $11 viz., $1,911, 9121,
progression. How far can you get? etc. No more than 6 people received the same prize. How many
- Contributed
prize winners were there, and how was the money distributed?
-Contibuted

Answer: 20 winners

248 249
po
CHAPTER 7 CHAPTER 7

2t
\ \N\

In the arithmetic of Puevigi, 14 is a factor of 41. What is the For what n is E k ! a square?
base of the number system? Contributed k=1 -Contributed

Answer: n = 1 or 3
Answer: 11

2SO
J 25L
CHAPTER 7 CHAPTER 7

5D,qD,
--
28
t
Find the only number consisting of five different digits which is No factorial can end in five zeros. What is the next smallest
a factor of its reversal. -Contributed number of zeros in which a faetorial can not end?
-Contributed

Answer: 87912
Answer: 11

252 253
CHAPTER 7 CHAPTER 7

p1 25

One is the smallest integer which is simultaneously a perfect Barnie Bookworm bought a thriller - found to his dismay,
square, cube, and fifth power. What is the next smallest integer Just before the denouement a lascicle astray.
with this property? Instead ol counling one through ten, a standard cure for rages,
-Contributed He totalled up the number ol the missing sheal ol pages.
The total was eight thousand.and six hundred fifty-six.
What were the missing pages? Try to find them just lor kicks.

'.i -CONTRIBUTED
j
I
t
t
f
{

254 255
ry-
I

I
I

i
i

CHAPTER 7
CHAPTER 7

P6 27

The reciprocals of the divisors of six sum to two, i.".,f +f The Sultan arranged his wives in order of increasing seniority and
1l
+T+?: 2. Find another number with this property. presented each with a golden ring. Next, every 3rd wife, starting
-Contributed with the 2nd, was given a 2nd ring; of these every 3rd one start-
ing with the 2nd received a 3rd ring, etc. His first and most cher-
ished wife was the only one to receive l0 rings. How many wives
had the Sultan?
-Contributed

Answer: 9,842 wives

256
257
I

CHAPTER 7 CHAPTEB 7

PE Dg
I'lI

If yo! solve the alphametic WATER - HEAT: ICE, you will The above alphametic involving Roman numerals is correct. It
double riddle: ..This bird,s ussurid of his
have the solution to this will still be correct if the proper Arabic numerals are substituted.
breakfast/and these before steeds cause a wreck fast.,, Curiously, Each letter denotes the same digit throughout and no 2 letters
70243 is the answer to both riddles ! stand for the same digit. Find the unique solution-Contributed
-Contriburid
f"' dlB
ftp' r*
\E Fry,q1
.€5*q \i
'{ itu#{*."q
L rJ rr iws -f P"
,td.eo'}
1

rp t,.)
. L --.;
!r
Hfur ;'"1:
*;***ry
ffi-,qffi
i

i
I

,fv
, ,
a_"".:,.
L-. i t'' ..

.J fl\
' .'
r.,. 1$
-''
t .-..:'

t
i-... r%
*i- llfl
o)
+.{

I lrsX
1{.
S"ffi
$e;dr

259
t"ffi*We
258
T
i

CHAPTEB 7
CHAPTER 7

80 BI
@o@@@@o
o@@@@@@
@@@@o@@
ru@E@tr@E
Find a permutation of the numbers one through seven with the Using a desk calculator, a student was asked to obtain the
property that when placed in both the first and third rows, the complete factorization of 24,949,501. Dividing by successively
seven rorv totals will all be perfect squares.
-lontributed incrqasing primes, he found the smallest prime divisor to be 499
with quotient 49,999. At this point, he quit. Why didn't he carry
the factorization to completion?
-Contributed

260 261
"T
I

CHAPTER 7
CHAPTER 7

B2 a8

Among those numbers whose literal representations in capitals The numbers 6,227,02O,80O; 6,227,028,000 and 6,227,280,000
consist of straight line segments only (e.g. FIVE), only one is are all large and roughly in the same ball park. But only one is
"orthonymic", i.e., is equal to the number of segments which equal to 13! Find it without use of tables, desk calculators, or
comprise it. Find the number. hard work. Contributed
-{ontributed -

Answer: 29

262 1
263
-"''lo* .
i

l'

CHAPTER 7 CHAPTER 7 I

t,

81 85
l

It

otra g/;h
TAO
ovotr
ftis rumored that the above inscription appears on the purple A certain magic square contains nine consecutive 2-digit num-
moon boulder, a fragment of which was brought home by our bers. The sum of the numbers in any line is equal to one of the
Apollo 11 astronauts. If the visitors who inscribed it were huma- numbers in the square with the digits reversed. This is still the
noid, and if the plausible inference is made that it represents an case if 7 is added to each entry. What is the number in the cen-
addition in a place notation system, can one make a further ter square? by Walter Penney
inference as to the number of fingersthese visitors had? -Contributed
Contributed by David L. Silverman
-
Answer: 17

264 265
lTTORreADYAIUCEI)
PBOBI-,BITIS

A eolleetiom,
of problerns frorn
aarioe,oa fields which
require eulsalus or
other solrhi,stieated
rnatherna,tieal tools

267
*4.&
-T
l

A ball is dropped from a height of 10 feet. It rebounds one-half the


distance on each bounce. What is the total distance it travels?
-Ma!hemalics MagaTine

Answer: 30 feet

&t&V?fiIT

269
CHAPTER 8
CHAPTER B

Which is grealer e r or r e2 *Conlributed Obviously the smaller the compounding period, the greater the
interest. How much does one dollar amount to after one year at
70O7o per annum interest, compounded continuously, i.e., in-
stantaneously? *
-Contrihuted

270 27l
CHAPTER 8
CHAPTER 8

Find the sum of the infinite series 1 ** ** +{ +{+ i** n Evaluate the infinite product II 2+t
2"+2
"4 3 59
610
.. . . , whose terms are the reciprocals of positive integers which -Contributed
are divisible by no prime ) 3.
-Contributed

Answer: 3 Answer: 1/2

,.7' 273
CHAPTER B
CHAPTER 8

A rectangular box without a top is to be made from a sheet of A student studying series starts with the familiar I + y2 + 1A +
metal in the manner familiar to all calculus students, i.e., by cut- ys + ... and inserts terms midway between these, obtaining 1 *
ting out squares from the corners and bending up the sides. The 3/c I yz + 3/s + 1A + .. . . He divides this by 2, since, as he ex-
finished product is to have maximum volume and its dimensions plains it. "There are now twice as many terms as before." He
are to be all integers" How will these dimensions compare if the repeats the process, interpolating terms between those already
metal cutout amounts to lUVo of the original sheet? -- Con tributed placed, again dividinCby 2. If he continues this indeflnitely, what
limit will the series approach? -Contributed

Answer: 6:3:1

Answer: 3/2

274 4t;t
CHAPTER 8 CHAPTER 8

('k,,)(""-,*)
Mr- X veers to the right when he walks. The curvature of his Archimedes O'Toole was so overcome by the favorable resPonse
path is proportional to his latitude. He starts walking North from among "Poeticians" to his last mathematical limerick (# 1-41 ), that
point A on the equator, in the area of a large level plain, and he composed another based on the above identity. Can you re-
finds he Is proceeding East when he is one mile noith of the construct the limerick?
equator" He continues walking and arrives back at the equator -Contributed
at point B. What is the straight line distance from A to B?

-Contributed

Answer: 1.2 miles

276 277
CHAPTE.R 8
CHAPTER 8

IO TI

Which contains more terms: the general polynomial of tenth


degree in six variables or the general polynomial of sixth degree in
Consider the sequence O, 1,2,7,20, 61,. . . . in which A,*,:3A,,
*2Aur,.Assuming the ratio of successive terms approaches a limit
ten variables? Contributed r, compute r.
- -Conlributed

Answer: r = 3

278 279
,j

CHAPTER 8 CHAPTER 8

T2 T8

The decimal .1 in base two equals.5 in base ten. Likewise.l2 in A boat owner agrees to take a group on an outing at $4.50 apiece if
base three and .123 in base four equal .556 and .422, rcspectively" the number of passengers is equal to or less than his bretk-even
Continuing in this manner, as the base increases, what is the lim- point. For each person above this he reduces the fare for all passen-
itrng value of the decimal? gers 3 cents per person. If he has on board now the numberof pas-
-(6n1yj[y1scl sengers that maximizes the total collected, what is the boat owner's
profit? Conlributed
-
Answer: 0 or 0.12345
Answer: zero

i
I

2BO I
28t
&,
CHAPTER 8 CHAPTER 8

t1 t5

A novice in calculus was required to differentiate an expression of The price per cubic inch for plantinum trays is the same as
that per square inch for platinum sheets. A metal supply house
the form Ax and evaluate at x:3. Naively using x Ax-l as the
has a square of platinum which will yield the same amount
derivative, he nevertheless obtained the correct value. What was
whether sold as a sheet, or fashioned into a tray of maximurn
A?
-Conyibuted volume with the four cut-out corners sold as sheets. How big
is the square?
-Contributed

Answer: 2.8564
Answer: Exactly 1 ft.

2B,2 283
CHAPTER 8

r6 AITSWTRS
TO
PBOBLEITTS

Prove that the digital sum of the product of any prime pair
(except 3 and 5) is 8. 4ontibited

2A4
c 285
,l

Ansuters
I
I CIIAPTER I
i

No; he has already used 4 minutes, the time that he has to go


the whole 2 miles.

q, He could put his gun diagonally in a cubical bor, 1 yard on a


raJ side"

An infiniteseries need not be the means of solution; a trivial


means exists. 300 miles.

The journey can be completed by starting it at an infinity of


points. One such point is the north pole; all others lie in the
neighborhood of the south pole and require one or more corn-
plete westward trips about the pole.

o> Yes. The first player should appropriate the only unique point
by placing the first cigar vertically on its flat end over the
center of the table. From then on he can counter each of his
opponent's moves by "reflecting" them through the center of
the table.

He divides the bricks into 3 groups of 3, 3, and 2 bricks. Then


he weighs the 2 sets of 3 bricks against each other. If they
balance then the heavier brick is in the group containing 2
bricks which another weighing will simply tell. If they do not
balance then he weighs 2 of the group that was heavier on the

247 Answers I ChaPter L,


15IY'

first weighing. If these 2 balance then the heavier brick was


set aside. If they do not balance, then the scales tell which is
the heaviest brick.
I

74 N if we consider the letters as the


four, flve, six, seven, eight, nine.
flrst letters in one, two, three,

I
I

The weights are opposite immediately after 3 and 9 o,clock,


a.m. and p.m., so Maynard must retire at 9 p.m. or 3 a.m.
I

i 75 The first group consists of numbers written only with curves


(in a certain type stfe), the second group consists of numbers
written only with straight lines and the third group consist6 of
numbers written with both straight lines and curves. Therefore
15 and 16 would be in the third group and L7 in the second.
Very easily. He takes the center square and then counters each
of his opponent's moves by taking the diametrically opposite
square.
76 25. In order to avoid pitching the last half of the ninth inning,
Hi had to be on the losing side. Thus he must have allowed at
least one run, which would hhve required at least one pitch.
,Among
many solutions we offer one: number the square in 24 more pitches were necessary to produce 24 outs.
horizontal rows as follows and start the ,,Knight,s Torlr" with
the square numbered 1, proceeding in serial Jrder: 1., 40, !3,
26,3, 50, 15,28;24, 37,2,39, 14,27,4, 49;41, 12, ZS,5g,
51, 62,'29, 1.6; 36,23,38, 6L, 54, 57, 4g,5;1,!, 42, 59, 56, 63, ,"17 2, 1, and 3. The sequence represents the number of chimes of
a wall clock which strikes once on the half hOur.
52-, 77, 30; 22, 35, 64, 53, 60, 55, 6, 47; 43,'1.O, 33, ZO, 4.5, g,
31, 18; 34,21,44,9,32, 19,46,7.

ra Two. The 4th and the llth.

70 Dr. LaRouche was buying numbers (for doors, gates, etc.)


the price was 10 cents per digit.
and

19 20 is in A, 2l is in B. Set A contains numbers with an eyen


(B with an odd) number of letters in their literal representation.

1001! + 2; IO01! + 3, .
77 1001! + 1001. n! * A is di-
visible by A as long as A > 1 and ( (n * l).
20 Without safeties, only one answer is possible: 0,3, 6, g. The
3rd period score could come either from a touchdown or 2
fleld goals. The 4th period score had to come from a touch-
12 No. Removing opposite corners leaves 32 squares of one color
and 30 squares of another color. However, each domino covers
one square of each color.
down plus a 2-point conversion.

Align the hour hand with the sun,s azimuth, and south will
21 Using "eyeJevel" test, he divides 54 c.c. between two test tubes,
returning Z7 c.c. to the beaker. Then he divides 27 c.c. arnong
7€t midway between the hour hand and 12.
be the three test tubes, returning 18 c.c. to the beaker. Next he
divides 9 c.c. among the three test tubes, returning 6 c.c. to the

Answers I Chapwr I 2BB 249 Answcrs I Chaptdr I


beaker. Next he divides 3 c:c. among the three test tubes, re_
turns L c.c. to the beaker, and combinls contents of Z test tubes,
leaving 2 c.c. in one. Finally, he measures out 2 c.c. from the 28 Any counting interval from 1 through 9 will result in another
team member's selection. Joe will be captain if he chooses 1.0.
beaker to an empty test tube; leaving 5O c.c. in the beaker.

!2

5) 5)
frr?
6 : 31, aG : 1., ale = 1. where or is the standard notation
for one of the complex cube roots of one.
29 ys1 S - -vfT
the other hand,
. If A is rational, it is the desired example- On
if A is irrational, then A,/; - 2 is the desired
example.

23 It can be done with eight colors, one for each row, since a
bishop move always involves a change of row. That no smaller
30 There are flve columns of cities. Since cities in odd-nurnbered
columns connect only with cities in even-nurnbered columns
and vice versa, a tour must alternate between cities of each
number will do is seen from the fact that the main diagonals
can have no color repeats. type. But there are ten cities in even-numbered columns and
twelve in odd. Hence a continuous tour is impossible.

21 He must gjck up 7 shirts to tide him over until the following


Monday. Ilence he must deposit 7 shirts each Monday. Countl
ing the shirt he wears on Monday, the required toiut i. t5.
31 This number holds true regardless of the manner in which the
ptzile is assembled. The proof is trivial. We start with 10O
pieces and end up with a single cluster. Each move reduces the
(Note that he cannot get by with only 14 by exchanging his total number of clusters by one. Hence 99 mov€S.
Monday shirt for a clean one and turning it in to tfre iaridry
as he will be caught short the following Monday.)

32 The misspelling of errors and receive are, of course, the first


two errors. The third error is simply that there are only two
errors in all.
Included among the 18 children were 8 married couples.
25

26:::,ff 1"I;::i"'1',;'#;:i:":;:Llf::l',iJ;il,1H?,L::
33 Not at all. Every closed boundary must contain at least one
pair of perpendicular segments forming an L. The 2nd player,
therefore,.can avoid defeat by completing each of his opponent's
and was therefore 100 times its original height. potential L's, drawing the foot whenever. the 1st player makes
a vertical connection and the upright whenever he makes a
horizontal one.
27 N_o. For example, Tom could have shot 3,4, and 5 followed by
15 4's, Dick 4, 5, and 3 followed by t5 4,s and Harry S,
3, i,
followed by 15 4's, in which case Tom would end t upon Dick, Between volumes 10 and 3. Evidently the librarian is shelving
Dick 1 up on Harry, and Harry 1 up on Tom. €t1 according to the alphabetical order of the volume numbers.

I Chapter I 290
291 Answers I Ch.aptei 7
-!n"*rr"
ti
Ii
;l

Each of these numbers (as also with "vief in Germqn,,or


€t5 *tpVl" in Russian) is equal to the number
of letters in its narie.
Moreover, they are tlae only numbers ,in their ,respective lan-
47 A dozen, a gross and a score
Plus three timesrthe square root.of four,
Divided by seven,
guages with this property. Plus five times eleven,
Is nine squared and not a bit more.
If you concluded that a one-move mate was impossible, you
€i6 failed to notice the orientation of the board. Since each pliyer
12 International chess law$ provide that when neirler K nor R has
has a white corner on his right; they are moving horizoitaTly- moyed and there *" ni obstructing pieces, K may move --2
Also, white is not moving toward the left. since he would already squares toward R while R occupies the tquutt over which
K
have the black king in check prior to his move; Hence white pi.r"., provided K is neither in check nor passes. lhrough-a
is moving left to right and the'promotion of his queen,s pawn in."ut"r"a square" Therefore: 1' Pawn is promoterl to roo.k'
to a knight results in a discovered double checkmite. Now regardless of black's response, 2. While castles on the
king's f1e! Checkmate. Purists^take note: the promoted rook
has-not moved as it is on its "natal- square- Blarne, not us'
but the rules which permit this esoteric loophole'
37 From any vertex r at least 6 lines of the same color (say red)
emanate, joining p say to A, B, c, n, n, and r. If any connection
among these is red, a red A is formed. If not, 3 of the lines en,
AC, AD, aB, and AF are the same color (say the first 3 are white).
Then if BC, BD, or co is white, a white A is formed with vertex "13 d3',60'(',]',)
at e. Otherwise A scn is blue!

a8 Assume the site is not Chicago. Pair each non-Chicagoan with


14 Turning! (as on a lathe). A cube with sideD can be turned
down to a cfinder of diameter D. This can be turned down
about an axii at right angles to the first, and lhe resulting solid
a Chicagoan. Moving the site to Chicago will not inJrease the
further turned down about the axis normal lo the other two'
total mileage. In fact, since there are unpaired Chicagoans, it
Straightforward (if slightly tedious) integration gives the results
will decrease it. Hence Chicago is the proper site, regardless of s v " 2 't-^ l2 for the three solids.
the geographic distribution of the other members. 6D, - D, - 4, 5' '-V

3 I r ff"::l JJ Hl,i" J: ln,i$xll" ; "'i,ffi T :T: :i: J:" J"fr


2nd player marks as many as necessary to complete either an
15 . . . RB. can oniy be NEARBY and
is obviously ASYLUM. FWF JID EQO WO?
. LR ' ' ' rnust be
WALRUS. Then we can convert KVJZDC into ASYLU', which

L or a T of 5 squares. In either event, the 2nd player wins.


PYX (a religious vessel)'
16
10 By noticing the progression of the second digits, you might have
deduced that this is the sequence of reversals of 2-digit primes (x.y) is a member only if y is the nearest state capital to state
in ascending order. The last member is 79. 17 capital x.

Ans.oers I Chapter 7 292 293 Anstlr;etx I Ch-aPter I


*trcr-
l

1a Both lights are red during the lst and l2th seconds and never
match at any other time. The answer, therefore, is 1/6.
Arusuters
CIIAPTER 2
19 The mirror really reverses nothing but ..apparent polarity',, and
it does this in every direction. Hold your right arm high and
your left arm low, and the image will appear to do the opposite
vertical "reversal". Object and image are exact replicas,
-a for point, by orthogonal
point projection through the mirror,
with reversal of "polarity" in all directions. Colonel Downing.

A thoroughly inbred family-the physicist not only married his


5,o-::::l1J:ilH#;:ffiff '-,*H':1"'ff;n:XY**:Iffi;: cousin, but so did his brother and his sister-resulling in a party
of two, including the host.
ing has $42 loss. Assume one player got them all. Then (D-O)
+ (D-O) + D - (26-D) : 42, ar,d D : 17, so Dave took 4
hearts and the queen of spades. By similar reasoning, Arch,
Bob, and Chuck took in 4, 3, and 2 hearts, respectively. If the conclusion were not valid, then every tree'would have a
different number of leaves; and you'd run out oI leaves before
you ran out of trees. It's valid.

51 It's love, violets, vile sot, I've lost.


The 2nd wife underestimated her husband. She knew the first
"What is so rare as a daY in June?" 14 eliminations would occur at positions 3,7,8,9, L0, Ll, 15,
5 2 20, 22, ?:3, 24,26, 27, and 30, but he knew thc first counted
out of 16 would survive 15 eliminations.
.l

"There are 3 ways to pronounce 'slough'."


53 in Philadelphia, fathers rarely
Seven-year-olds don't have wives
confusl their children's names this kind of logic plus a
51 Mudvi-lle got 30 straight walks. Casey, batting ninth, was picked
off first base while bowing to the southside bleachers after his
initial walk, and was picked ofl third the next time while doffing
little digging reveals that Mr. White is the presiden': Yt'
Brown [- tne protessor, Mr. Black is the instructor and Mr' rl
Green is the Janitor. tr

his cap to the fans on the northside. While bowing to the um- i
pire after the 30th walk forced him home, he neglected to
touch home plate and was declared out in the dugout. .l
The first student will have said "No," whether or not he is
studying liberal arts. Ilence the second must be telling the
trutfr, and hence is not in liberal arts' If the third is not in
liberal arts, then the first is, and vice versa' There is only one

Ansuters I Chapter I 294 295 Answers I ChaPter 2


liberal arts student. If the third is in liberal arts, then he,s lying pair of them are acquainted, that pair and Srnith are mutually
and the first really is not in liberal arts. But the word ..really,' acquainted, and if not, A, B, and C are mutually unacquainted.
implies he said he was a liberal arts student, which is impos- The other case is handled symmetrically.
sible. Hence the third cannot be in liberal arts, and thus the
first is"

A
observes that B and C are marked. A reasons that if he also
72 3.7 dollars. If the total weight amounts to 60 ounces, then the
grocer from whom I did not Purchase a packet gives short
weight. If the total weight is 593/a ounces, then the Srocer
is marked, B and C will each see two marked foreheads and from whom ,I purchased one packet gives shorl weight, etc-
all hands will stay raised. If, however, A is unmarked; B and C
will each see one marked and one unmarked forehead. B's
hand will be raised only because he sees a mark on C, and vipe
versa. Very shortly either B or C will recognize this state of
affairs, deduce that he is marked, and put down his hand. A
73 The distance from Kroflite to Beeline must be at least 25 miles-
The towns could then be located at distances 0,1.,4, 1'0,-18,23,
and 25 miles from Kroflite. There are 21 distances between
allows enough time for B or C to come to this conclusion, and towns and these are all distinct. Any shorter distance would
since neither does, A knows that he also must be marked. mean at least one duplication.

Gwen loves Alan.


71 R G R B R G R. He now cannot use R for lhen there would
be two consecutive R's. He cannot use G'becnuse there would
be two consecutive R G's. He cannot use B for then there
It does make a difference. A submerged body displaces its would be two consecutive R G R B's. Any other pattern for
volume; a floating body displaces its weight. Since a penny is the first seven stones would have allowed a choice for the
denser than water, dropping it into the canoe will raise the eighth not involving a repetition.
water level higher.

To insure a win the first player must subtlact .g' He then


10 If we assume B is telling the truth, then by following the im-
plication of his statement we find that D is also telling the
15 counters his opponents' plays of 1, 4, 9, or 16 with 9, 16, 9,
or 4 respectively.
truth. If we assume B is lying, we find -that C and E are telling
the truth. In either event, however, A is lying. Thus A is the
only suspect we know with certainty to be lying.
16 Weigh
ones-
A against B, C against D, then match the two heavier
Without loss of generality assume A heavier than B and
C, with C heavier than D. This can be accomplished in 3

17 Call one of them Smith. The other 5 can be placed in


gories: acquainted with Smith or unacquainted with Smith.
Z cate- weighings. Now weigh E against C, and supPo$e E heavier than
C, in which case weigh E against A (5th weighing)' If A is
One o{ these categories contains at least 3 individuals, say the heavier than E, then match B against C, followed by a match
first, i-e., A, B and C are acquainted with Smith. Now if any between B and E or B and D depending 0n whether B is

Answers I Chapter 2 296 297 Answets I ChaPtei'Z


il

i
i

heavier or lighter than C. If E is heavier than A, match B versity of Chicago has no football team, l9-point cribbage
against C and against D if necessary, which completes the or- hands are impossible, as are solutions to the equation ger : 1,
dering in 7 or less weighings. The case E lighter than C can be since this implies e* : 0, which holds for no value of x, r€al
handled in a like manner. or complex. Thus each class is empty.

Strephon should pick one of the end petals from the group of qq Suppose otherwise and assume that there are n Irishmen. The
77 8 making two groups of 7 and 2. If then'Fhyllis leaves 7, he
can rnake this 3, 3, and if she leaves 2, 2, 4, he can make this
mm possible number of Irish friends for a given Irishman ranges
from 0 to n - 1, and each of these possibilities must be re-
2, 2, l, 1, in both cases winning since he can duplicate her alized in order that all n numbers be different. However, iI
later moves. If she leaves anything else he can convert it into one Irishman is a friend of all the others, no Irishman can be
l, 2, 3, and win. Strephon can also win by leaving groups of friendless. Thus the 0 and the n
- 1 are mutually inconsistent.
8 and 1.
Let T denote the set of citizens who drink tea and T, the ones
2€t
l A',J i i'ili:iH I:il1'"1",:;,f ;Ii l#"'JT*j.,fl'""i.'.1,';
ing with O 0 and 0 1 1. The only way to do this without
who don't, and so forth. Then T' contains lO% of Eristol's
citizens; C',20Vo;W',307o; and G', 4O7o. Since no one drinks
all four beverages, the union of T', C,, W', and G, contains
IOO% of _the citizens, implying that these four sets are disjoint
repetition is 0 0 l, 0 1 1, 0 1 O, O 0 O, 1 0 O, 1 1 0, and 1
in pairs. This means that every citizen partakes of three of the
1 1. The buttons he pressed were therefore, 1,2,3,2, 1.,2,3.
four beverages. Hence l0O% drink liquor.

7 g i:i:', :TT: i:11",::: ;r,lx' * i #: i,'I:,'i i:*"r"'"?:: 21 Mother.


If
If lst night, she'll say yes the 2nd.
she says no the
is asked the 2nd night. In either case,
she says yes, Father
2, 4 or 2,2, 3, Smith can leave 1, 1,2, and if Jones leaves 2, permission for 2 consecutive nights out rests with hirn. By
5 or 1, 1, 5, Smith can leave 1,2,2 or 1, 1, 2, respectively, in svmmetry, success would depend on Mom if Dad were asked
all cases winning on the next move. lst. Since Dad is more permissive, Mom should be asked 1st.

20 The first question should be: "Will your answer to my second


question be true?" If he answers "No," he is a Diplomat. If
not, make your second question: "Are you a Diplomat?" If he
25 Bidding either 910 or $9.99 guarantees you a profit of at least
$1O, but the $9.99 bid is superior in that it rerults in the same
profit as the $10 bid in every case except those in which your
says "Yes," he is a Dissembler; if he says "No," he is a Sooth- friend bids less than $9.99. In such cases the $9.99 bid nets
sayer. you an additional penny.

Penguins are confined to the Southern hemisphere, peahens


21 take care of the egg-laying. Hungary is landlocked, the Uni- P6 The statements cannot all be false; that would lead immediately
to the contradiction that the last statement is true- On the

Answers I Chapter 2 294 299 Answers / ChapterZ


-

other hand, not mofe than one statement can be true. It follows
that statement 196":l is true and, all'the rest are false.
Ansotter's
CEAPTER 3
27 Yes. At T:0 start both timers and put the eggs on. At T:11
invert the 7 minute timer. At T:11 invert the 7 minute timer
again! It has been running for 4 minutes, so it will run out
again at T=15 at which time your eggs will be ready.
He lost, even if they played only twice, or four times, or six,

pa Since each school played ten league games, the other five teams
won 2, 3, 5, 6, and 8 games,. respectively, or a total of 24. The
or ..

total number of league games played is easily determined by


imagining that every team wins 5 and loses 5 for a total of 30
If the prisoner places one white ball in one box and the remain-
ing balls (9 white and 1O black) in the other box, his chance of
games won and lost. This leaves 6 wins for Hard Knox which,
survival would be (l + 9/19)/2 = 0.737, or 73.7%.
therefore, tied for second place with .600.

The first man's odds


'..3 are

+.1+ ;"+ + +; l+ :+( ,*i *L*


*
i )= *(:)=i
Let A, B, C, D be any 4 of the points. Let them be so ordered
that ABCD is a polygon. Then the lines AC and BD
are uniquely determined and they form one intersection in-
side the circle. To each set of 4 points there corresponds
a unique intersection within the circle. Hence, there are C,,4 or
n(n-1) (n-2) (n-3)
-- 4-:;:-- such intersections'

There are 4! or 24 possible permutations of 4 cars. Only one


of these is in increasing rank of licence magnitude. Thus there
is one chance in 24. A\e number of cars in the lot (999) is
irrelevant.

Ansrners I Clwpter 2 300 k 301 Answers I Clwpter 3'"


I

The probability is one, since any three points on the 'surface


of a sphere are always located on some hemisphere. 70 Yes, by considering the throws
currence
in pairs and tleleting each oc-
of HH or TT. Since IIT and TH are equiprobable,
one may be used to denote 0, the other, 1.

Let GA and BF denote the respective events: girl baby ar- Assume two white marbles have been drawn. From the condi-
rives at 2:00 p.m., boy baby selected first for footprinting.
Then P(GA) ' P(BF, given GA) : P(GA and BF) : P(BF)
77 tions given, the chance that the third marble is white is l/2, i-e.,
there are an equal number of marbles of each color at this
'P(Ga,givenGF)o.+.=+=+( 2 3 \ point.Hencew:b*2also
z g*3 2.\C+3+e+r-)
P(GA, given BF), where g is the original number of girl babies.
,=(;)=. : t/3,so rhat (2w-2)(2w-3)
- w(w 1)
Thus P(GA, given BF) - 2/5 and, is independent of the num-
ber of girl babies. -qL:
(*,')- (,w-2)
l/3, giving w : 6; b -- 4.

The sub escapes if 4 depth charges miss (probability =


i
a 1

256)
or if 3 miss and one damages (probabitity - *).
4
Hence
72 If A marries B's sister, B marries C's sister, and C marries A's
sister,a group of three is possible. An additional mutual
brother-in-law, however, is not possible without violating either
the sub escapes with probability is with the laws of bigamy or consanguinity.
ability 251
fi "na
sunk prob-

256' If the probability that a team will win any particular game is
78 p, the chance of winning the Series is pa * 4pa (l - P) t
tOpn 1t - p), * ZOp+ (l - p)a. If this expression is equal
to .4,p will be equal to .4539 approximately. Ilence B's chance
There are (?) or 715 ways of choosing 4 cards in a suit, of winning the flrst game is .5461. Since odds of 6 to 5 cor-
respond to a probability of only .5455 approximately, the
(T) - 286 ways of choosing 3 cards in a suit and (1j) gambler is on the safe side.
or 78 ways of choosing 2 cards in a suit. There are, in addition,

f=+=) ways of assigning the suits in a 4-4-3-2 hand,


\2! 1! 1!/
and f+i)
J

ways of assigning the suits in a 4-3-3-3 hand.


11 Each lap after the first contributes one degree of uncertainty
to the salesman's location. If there are N cities, N - 2 laps of
N - 1, plus the flrst lap of N, plus one more city of the (N -
\3! l!/ 1)st lape will be necessary in order to make the salesman's
The total number of hands having these distributions are, there-
f.ore, lZ(7 15;2. 1286) . (78) and 4. (715) " (2S0)e respectively, whereabouts completely indeterminate. This gives a total of Ng
or 136,852,887,600 and 66,905,856,160, more than two to one - 2N + 3 days or 83 days when N : 10. (On the 82nd city.)
day
in favor of the 4-4-3-2 distribution. it could be definitely stated that he was not in the first

Answers I Chapter 3 302


303 Answers I ChaPler 3
75 Let h !e the probability of getting heads in a single throw.
Then h2 - I - h and h is approximately 0.61g.
results in a decision is 2/3, and the expected number of repeti-
tions required to determine a loser ,,
:' :
l'/r.
o!, ?- (+)-

* (:) possibilities, only the 3 rows, 3 columns, and 2 diag-


7 6 There are 4 choices of wife for the oldest of the 6 boys. If his
onals constitute a win. The answer is therefore, z P7 brother marries hi+ wife's sister, 4 distinct pairings of the other
2L' 8 people are possible. If not, 16 pairings are possible. The total
is then 4 (4 + 16) - 80.

17 Let 1, 2, 3, and 4 denote the amount of the bids in increasing


order and consider the 24 possible permutations. The homel
5),5), (10 - 2) (1o - 8) (10 - 1) - 8 * L36 possible codes'
owner will optimize his chances by looking at the first bid and NJ fiJ
using it as a standard, i.e., by accepting the flrst subsequent
bid, if any, which is less or the last if- none is smaller. His 1

chance of accepting the lowest bid is easily seen to be ll/24.


If he uses the l6wei of the first two bids as'his standard insiead,
then his chance is reduced to lO/24.
23 Normally his chance of firing the fatal bullet
man spinning the chamber, the probability that the first flve
is i. With each

men survive is (*): ""a his chance of being shot is f "t


that or about .067. Hence his survival probability is enhanced
r8 Let the fraternity have x members. Then (;) :
(. ;
,),
leading to a quadratic equation with roots x :'2 or 't5. Only
about .1 by spinning.

the latter meets the conditions of the problem. Thus the fra-
ternity had 15 members and could field 3,O03 teams of either
type. 21 of the (aj) arf*"nt
(T) alu'*lro
pairs of cards he could draw, there are
pairs and 16 pairs each of type 2-3, 2-4, 3-4'
i-t,'+-l , 4-8,7-8,7-9, or 8-9 which yield straights- One of
79 There are altogether 73 or 343 possibilities. Of these, 49 read
the same backward or forward. Half the remaining 294 must be
eliminated since they are duplicates. There are, therefore, a
each of the latter consists of 2 -diamonds, i.e., has already
been counted. The probability of a pat hand is therefore,

total of 196 pieces in the set. + ( 1s) (e)


or about .168.

20 On a given match, the probability that all three coins come up


the same *, (+)' : !, while the probability that alr are dif-
of 8 into 3 numbers are (6,1,1), (5,2,1), (4,3,1),
terent rs'3
2t2
3- -9' Hence the probability that a given match
25 The partitions
(4,2:2), (3,3,2). Each partition involves either a duplication or
2 numbers with a sum of 7, neither of which can occur among

Answers I Clwpter 3 304 305 Answvrs I ChaPter 3


3 faces meeting at a corner of a correctly spotted die. Con- the finite series: 9+ (9)(9) + (9)(9)(8) + (9)(9)(8)(7)
sequently, the probability of making 16 the hard way is zero. (9)(9!), which by successive cancellation can be
+'''+
written:9(1 + 9(1 + 8(l +7(...(1 + 2(l +1))))))))).
This is readity evaluated to be 8,677,690, the nurnber of posi-
If Max throws doubles, his chance of winning i. (l)'. If ni, tive integers all of whose digits are distinct.
26 two dice come up di.fferent, his chance * fi)i tilror'h,, ,orr,
probablity i, (;)' +-; (A' :
* H - .)"r{. Bermuda, with ot ,n"(i1)possible hands, there are exactly 4 in whibh all
a probability of .514 is the betting favorite. 37 cards of a'suit are held (one for each suit). The maximum
point count involves 4 aces, 4 kings, 4 queens and a jack. Since
ihe iact< can be in any suit, there are also 4 vays of achieving
this 37 point hand. Thus the two events are equally improbable'
27 If V. 4 is just right of V. 1, he eats through no pages at all;
there are six such cases if we imagine the books arranged in
all possible orders. Likewise he eats through l, Z, 3 or 4 books if
the volumes are arranged in any of 4, 8, 4 or 2 ways, respec-
tively. (He will eat through 4 books, e.g., if they are arranged 32 Let B denote any of the 35 suit or no-trumP calls; T, 2 passes
in a row; D, double; and R, redouble. Then the sequence
BTDTRT consisting of 9 bids can be repeateil 35 tirnes for a
423 I or 4 3 2 L.) Therefore,
total of 315 bids. An initial 3 passes plus a terminal pass bring
Ep:300 :500. the maximum total to 319. (Some readers rnight argue with
some merit that the final 3 passes following the redouble of
seven no-trump are academic, in which case the actual maxi-
mum would be 316.)
28 With four specimens, the odds in favor of a mated triple are
or;ly 4/9. But if payload limitations permit five to travel to
Earth, the odds go up to 50/81.
Your team wins right away either with hit-hi1 (prob. 1Z times
iB€t Vz = Yq), hit-miss-hit (YB), or miss-hit-hit (t/e)- Total protr':
Vz.You lose only if all 3 shots are missed, with prob. r/a'The
29 In the long run, the three black balls will occur equally spaced
in the stream of balls which emerge. The player, therefore,
can expect three white balls to appear before the first black ball
remaining prob. of 3/s includes the 3 cases in 'which exactly
one free throw is made, throwing the game into overtirne' Your
team ratgs a Yz stake in this event, so your overall prob. of win-
and hence will show an average loss of a quarter a game. ning is 3/L6 + Vz - 11'/16 (about 69Vo).

30 There are nine such numbers of one digit. In the case of two
digit numbers, there are 9 choices for the 1st digit and 9 for
the 2nd. With three digit numbers, there are 9 choices for the
€t1 The probability of a loaded face turning up is 1/5, of a given
unloided face 4/25. The probability of the two loaded faces
turning up together is l/?5, of two unloaded faces adding to 7,
1st, 9 for the 2nd, and 8 for the 3rd. This gives rise to
16/625. Sin"" th.t. are 5 such combinations, the total chance

Answers I Ch.apter 3 306 307 Answors I Chu.Pter 3


*T

of throwing 7 is t/25 + I#P = Hence the effect of the


operator can exPect to make a profit of a nickel a Same on
* the average.
the loading is to increase the probability by o, ooty
# - |
one part in 750.
10 Every sequence which fails to qualify (call it ''bad") has a frst
poini at which blacks : reds. If the colors are reversed up to
this point, we have another bad sequence. Thus bad sequences
35 Since 417:3(5!)+2(4!)+1(3!)+3, the first digit has progressed
from I to 4, the 2nd from I to 3 and the 3rd from 1 to Z. fne
com" in pairs, one of which starts with a red, Since eYery_se-
quence tiat begins with red is bad, the probability of a bad
last 3 digits constitute the 3rd largest permutation of l, 5, and riqr".r"" is twice the probability of starting with a red or
6. The 417th term is 432516.
, (*J =fi, uoathe desired probabilitv is
,f,'

36 Prob. (L wrons, F right) --


+ i = i.
Prob. (F wrong, L right) : j.i :
+.
Ilence the odds in favor of rain are a/n b /6 or 3 to Z, and
prob. (rain) = sl.

37 The player's expectation, E, is Vz*4(Vo)+9(ya)*


/zB:Vz+3(%)+5(ys)+ . . , and, sub-
that, by subtraction,
tracting again, YeE=Vz*2(r/+)+2(%)+ . . , so that vaE=
, so

y2+1, or E:6. Therefore the operator can expect to clear four


dollars per game on the average.

5D <-f The series will end irt 4, 5, 6, or 7 games with probs. l/9, l/4,
arO 5/ft aod" 5/16 respectively. The 6th and 7th games are played,
if necessary, at the same park as the lst and 2nd games. The
desired probability is, therefore, 5/8.

39 The average number of empty cells when n balls are thrown


into c cells is If n : 5, c: 4, this becomes *
$s #,
that the player's expectation is approximately 95fi. Ifence the

Answers I Chapter 3 308 309 Ansuters I ChuPtet 3


--.i!E'

IL should take the tranas {f hours to come together, not 65


Am,sottera _
minutes or
13 .
fi horrts as
it actually does. Therefore, Dr' Reed's
CHAPTER 4 watch runs too fast by a factor of. (t2/t1' + L3/ll) :ffi'm
one hour it gains 60l(144/1'43) - 1l or 6o/t4l minutes'

8x-5(25-x) :0; 13x:130; x:10. Let T : initial position of the minute hand, in minutes after
the hour. tte initial position of the hour hand is therefore:
TT. (20 *
Qo + ;). The hour hand travels a distance 1- U)'
5) 413 (2X-3o) -541-72:48; 4(6X-144) :120; X:29.
rJ whichisl/lzrhthedistancewhichtheminutehandtravels,
T
60 - [T - (20 f ,)1, or:
Let T, - the setting of his clock when he left home
: the setting of his clock when he returned
Tz
Then T2-Tr : the total length of time he was away
r- (20 * il = 60 - tr- (20 + ;)1.
Let t1 = the correct time at which he arrived at his t?
3840
friend's home Solving, t = ffi - 26'853L ' minutes and the time is
tz : the correct time at which he left his friend's
home therefore 26.8531 minutes after 4:00'
t2-t1 : the length of time at his friend's home
Then t2*Vz [(Tz-Tr)-(tz-t)f is the correct time when
he arrived home, assuming he walked at the 5562 seconds.
same rate both ways. I
First there are 2O flags whose top and bottom rows are of the
764,488.
10 same color (5 choices for the middle row, then 4 choices for
the outside rows). Then there are 5x4x3 or 60 flags containing
Since every integer has a unique decimal representation, every 3 colors. But half of these can be obtained by turning the other
non-negative integral power of X will appear exactly once in half upside down, so the actual total is 20+30 or 50 flags'

the product. Hence the product equals ; x^ = 1/(1 - x).


n:O or from the floor'
The ladder will touch the wall 3'62 1'38' feel
77
Since each of v/wz, w3/x4, x\/yo and yt /za is a constant, there-
There are 80 steps in the escalator'
fore their product,
'
v w. x
z z z
y,
z' is a constant. 1p

Answers I Chapter 4 .3to 3lr Answers I ChaPte*^4


ilit
lir

ili
rll;

iil
ii
t:
lii
5 .5 ri

on a salary of 50 -+- X rupees, a Franistanean pays (tor**\ is negative. Ifence the fust two fractions were
5 and, m
7€t .(50 -t- X) rupees and is left with ,5
ni' \ 1oo / either order and the third-3.
rll

- ifu
rupees. Since ui

X = 0 maximizes the take home pay, 50 rupees is the optimal


salary.

ra In general, the maximum number is given by *y - x - y, where


x and y are the number of balls in a box. In the case considered

11 If the number of cities is


number of triangular flights
of the form 4n + 1, the minimum
is (?) . (^ r* ') - n. In
the maximum number is 97.

theproblem n:z.rherefore (i) .(;) - zor 4+r0 79 The two expressions are identically equal, respectively, to the
smaller and the larger of the two numbers x and y.
- 2 - 12 is the number of triangular flights.

15 If we let the distances the two snails travel be a and b miles


and the speeds (in mi./hr.) x and y respectively, we have
20 If Jones parks A hours and Smith 7A hours, they pay resPec-
tively X+
f faf X+and
+ [7A], where [A] denotes the
X : 3, y : b *o : :f +ff. u",." * * +.xy - y2 : o largest integer less than A" Since we are given that X *

? 4rtst
rrri.: : { x + +tAr }, 3 + 3 IAI - llAl,frornwhich
from which * : iy. The first snail, therefore, traveled at three-
A must lie between 3/7 and 4/7 i.e., between 25 5/7 min. and
fourths the speed of the second. 34 2/7 min. Since the time clock registers in S-minute intervals,
Jones parked for half an hour and Smith for 3/z hours.

76 Assume there are 2n + | pearls. Then the weight of the pearls


is 101n * 1. The weight of the clasp and string is 2n * 1, so
that the total weight is l03n * 2. Since this is equal to 1650, 27 25%o. Let speed of Mr. Field
speed limit) :
= V, speed of traffic flow (i.e.,
S, average interval between cars (in either direc-
n : 16. The necklace, therefore, contains 33 pearls, the weight tion) = A, number of cars overtaken (same direction) per
99 unit time : L^j, number of cars encountered (opposite direc-
lncrement berng
l6 carats. A v-s
tion) per unit time : af,. *.o 9
1

If the first fraction *"r" the second would have to b" "hu'av=is'
A
17 f, *
in order to make the result ambiguous. Likewise the third
would have ,o b" .lf a2 - ab * b2 = 19, one of 5)5) The number k is seen to occur first at th"(ry+ r)tn
;-fr--* 6, .t'J \ - /
the variables is 5, the other 2 or 3. But b + S; otnerwise position. Calling this latter expression n and solving for k,
;ft

Ansuters I Chapter 4 3r2 3r3 Ansuters / Chapter <l


*r
t
1
:

-R: 1+\/En-7 Let each expression: y. Squaring both sides, y2 = x -f


In general, by the nature of the progres-
---r---------:-.
sion, the nth term is the greatest integer S this expression.
27 xy. Hence Y : x, 2t : x2, and x : 0 or 2'
Y =

The millionth term is, therefore, t,41.4.

X - the value of the first expression and Y the value


2€i If C : number of questions answered correctly, B : number 2A Letting
of the second, we have X : 3 + { ana Y : 3 + }, t.o*
of blanks and E : number of errors, the score is C B
We have, then, Cp - Bp - ZF;p - C*
- - ZF,. which X = Y = 4. Hence both expressions are esoteric rep-
- Bon - 2E*. Since
Cp * Bp * Ep - CM + BM * Eu, this becomes 2Bp resentations of 4.
- 3Ep =
2Brr - 3Ey. Also, Ep : 4E* and Bp * 9 : By from which
EP : 8 and Eo, = 2. Pat, therefore, made eight errors and
Mike, two. qA Solving the inequalities simultaneously, we flnd that 56lza 2N;
flC' 5312. Knowing that N: 54, 55, or 56, the other inequalities
lead to the unique solution: P : 26, E : 19, N = 55.
A A'*h"." 4
24 we have log
logB= B B -
2
J
Hence3logA-2log
(+) rrom which 6 - 2, B = T.
27
The original expression
,€toH".H#'fi,;'i:[xt'1,H"',*3,111"T:l:"#l.if ::;r];
" ite hyperbolic equations cannot exceed 4'
solutions to a pair of
rog * ' Thus two hyperbolas can intersect in no more than 4 points'
was, therefor., --*.
ros
t1
+ 4 .VT)* - 4(z + VO" + 4 : 3, and since (7 *
37 (7
a/l)= - (Z + \/T)2 we have (2 + \/r2" - +(2 +
25 If X is the distance
between two neighboring towns and R the
rate of the 4th car, we have X2 + tX - (2R + 32))2 :
V)- + 4 : 3, in which the left member is a perfect square-
Thus [(2 * flf)* -
21a 3, or (2 - z * VI= - - *
\/T
2O42 and, X2 + [X
- (2R + 16)]2 : 2122, from which X : and rejecting the negative value, we obtain the only real root,
2R + 128 and substituting this in either equation yields R : 26
or R : - 154. This latter must be rejected, so that the flrst car --_t
l
-
l.

traveled 50 mi./hr. and the distance between neighboring


towns is 2 x 26 + 128 or 180 miles.

32 Itsuffices to prove it for two sacred numbers, since the theorem


then follows by induction. Let M : A2 * B2 and $ : C2 l-
D2. Then MN : (A2 + B2) (C2 + D2) - (AC + BD)2 +
qG, Using the rule that powers multiply by adding their exponents,
flJ 1, a multiplicative magic square is easily obtained from the given (AD - BC)2. Amen.
square by substituting 2" (or k" where k > 1) for n in each
block.

Ansuers I Chapter 4 3t4 315 Ansusers I ChaPter 4


-T
i

and fly. There are three of these, of length {X, VT' "o1
Am,su)ers r,/tilunits. according as the spider crosses the 1,2 or 3 unit
e"rlge; of these, the la"st is the iequired path.
CHAPTER 5

The angle u = ft satisfies the relation tan 3a = cot 2a, i-e-,


One cut in the third link will allow two links to be swapped
3tana-tan3a l.-tanza -or)xr- F 4
10x2f1=0,where
for a kiss and a link on the second transaction, and 3linki-for 1 - 3 tan2 a Ztana
a kiss and 2 links on the third and so on. x=tana.

58 feet. By unfolding the walls and ceiling into a plane'


it will
First calculate the times at which the minute and hour hands of a right
be found ihut ttt" bug travels along the hypotenuse
are exactly 120 degrees or Vz circle apart. Then calculate the
position of the second hand at these times, and choose those triangle with sides 40 and 47 feet'
times whqn the difference between the hour and minute hands
is closest to 12O degrees. The closest difference is 4/ll
Determine the height of the equilateral triangle to be 3
(.3636) circle at T : 9-l/11, hours and T : Z-L0/LL hours. So inches'
the answer is 2 hours, 54 minutes, 35 seconds. The only other The sum of the diameters of the circles at each vertex is 1
time could have been t hours, 5 minutes, 25 seconds. inch, and the sum of the diameters of all circles is 5 inches'
is 5
Since I zr d i : ,, 2 d i, the sum of the circumferences
inches.
"r
There are 21 edges. A famous formula of Euler says vertices
-
edges f faces : 2, for any solid figure with no holes, so that
The area of the triangle will be minimal if the beech
the number of faces is 14. tree bisects
the side on which it lies. The area of the triangular portion.is
g39,l2} square yards. The length of the fence is 2O18 yards'
Let AT and BT be the tangents, ACB the arc, O the center, OA
the radius. Let angle AOT = x and AO :r. Then arc ACB :
(2o
- 2x)r:1, and tan x : 1/r; whence tan x: Z(rr x). Draw two lines through the center of the square' perpendicular
By approximation: x : 74o 46.2,;
feet/tan x : 1437.45 feet.
r : 1 mlle/tan x : -52BO 70 to each other, such tlhat each is parallel to a side of the untt
square. These two lines partitionlhe unit square.into t* .?
the
,.rit .quu."r. At least Z at ttte 5 points must be in (or on
points
perime^ter of) one of these smallei squares' This pair-of
Imagine the walls, etc. of the room to be made of cardboard cannot b farther apart than the length of the small square's
rectangles, which are then "unfolded" into a plane. The shortest
path must then be one of the straight lines joining the spider diagonal,
-f1
r-- untts.

Answers I Chapter 5 316 317 Answers I ChaPter 5


rY il!

i[|

that a : 3\/5. The truck will therefore be able to go under


77 From P draw a line, L, to the opposite vertex, say A. Now
construct a line parallel to L from the midpoint of BC, inter- an arch 6/5 or approximately 13 ft' 5 in' high'
secting the side of the triangle at Q. The line PQ divides the
triangle into two equal areas.

The distance will be a minimum if the two legs of his trip


make
7A equal angles with the stream- This will be the case if he reaches
thl strealm at a point 2/z miles downstream frorn the point
I{e
7P t/-Z .
arrives at the North Pole having traversed a distance of
10? meters. directly opposite. The total distance will then be 8y'-5 or aP-
proximately 17.9 miles'

Exacfly 600 yards. It


can be shown that in any general tri-
7€' it in two equal areas is equal to Assume another man is also approaching the building
from the
angle the shortest line dividing
VZ-Is - a)-G --6), where s : semi-perimeter, and a and b
are the two longer sides. The fence will terminate at the two
79 diametrically opposite direction' If the flrst rnan can
sides, the ,"torrLl-*un can see three sides, and lice
versa'
see two
There-
be the same as the
longer sides. fore the chance of seeing two sides must
chance of seeing three siies and since only two or three sides
-t
canbeseenatonce,eachoftheseprobabilitiesrnustbeone-half.

11 Exactly 100 yards. The reasoning is that at any moment the


hares are at the vertices of an equilateral triangle which di-
minishes at the velocity of the hares. Henc" $r. the time it
V
takes for the triangle to become a point in the middle of the
20Tliliestotheeastoftheothertwo,thetleasurevillcertainlylie
jy,.,#:'i"'l'*'#Tu'.f*'J"ff ;':,i::ii,n5',ill:'ifl:

field. The distance the hares travel during that time is VxI outside the 'tree-angle'' Otherwise' since the angle opposite
V
the 26 side is arctan rcn"" greater than 60o' whereas the
which equals exactly 100 yards. ff, 'will lie
trees form angles of 60o with each other' the treasure
outside the trEe-angle also' Therefore' regardless of.
the orien-

15 The smallest integral solution satisfying the given conditions is


a: 25, b : 16, c: 39.
tationofthetrees]thetreasurewilllieoutsidethetriangle
they form.

The two sheep have exactly one acre to themselves.


was 24 inches wide. The creases met the top and bottom
at
7 6
21 It
points 5j and lsj inches from the corners' The perpendicular
4,
5 " # : 1 as the equation of the ellipse, where
ltu distance between the parallel lines was therefore inches or
We have *
1J 2a is the height of the arch. From the data, x - 3, y : 12, so of the width.

319 Answets I ChaPter 5


Ansuters I Chapter 5 3tB
qfl

qq
nJ /1-
The two cities and the oasis form a right triangle with hypo-
tenuse 75 miles. Letting d be the shortest distance from the
the cube of its height, ro y! =fioy,and V :#-
the relative distance from the apex of the "3
cups"
Letting a =
to the railroad,
# by similar triangles.
oasis Thus v
*: mark, l- und x ,- .51. Thus the "3 cups" mark is
d = 36 miles, requiring a two day trip. f,Il: xi
(The
about ZVo of the way down from the top ol the pot'
capacity of the pot is exactly 3r/3, cups')

23 Half the length is a side of a right triangle with the other


3x x and..hypotenuseT.
3x
side
the sixth at the
s - 2*
rJffi
2a Place five at the vertices of a regular pentagon'
center of the pentagon, and the seventh above
distance equal to the radius of the pentagon'
the center at a
Therefore x - 8 from which
8
or approx. 20.69 inches.
1-LtfT. 29
The weights are proporti""l t. the volumes' The volurne of
T P, wnere r is the ladius at the top'
the original icicle was 7
The volume a few hours later is *no'r' where R is the
1a2 cx-z r b2 qx -z: (a2 * of the
21 ;;)*"5::1',"JJ,*::'-.0--' radius of the larger icicle. If O is the generaling angle
original icicle, R = 20 r tan 2 O' tan O =*,
"
,.7
25 All three are the same size, each being equal to
R is the radius of the circle.
*, R2, where
R:20, ---7TV: ff.,
-10 to that the ratio of the two
, - \,0/
volumes is 2 (H)' ately 32'65' The new icicle'
26 The required area will be a diamond-shaped figure composed
of two triangles with a common base, each triangle similar to
the original triangle, but with each side only one-twelfth as
or approxim
therefore, weigtrs almost 33 times as much as it weighed before'

of i,
lj, .o that the total area in
large. The area each triangle
She lays off equally spaced marks around
the oerimeter of the
which the pin may be placed is 2Vz sqtare inches. €io cake, the number of marks being equal to the
irumber of cake-
mark to the
;;;;;; ilJ;, and makes a vert[al iut from the top
each
areas are
c"nte"r'ot the cake' By elementary geometry
are also
27 The pot is the frustum of a right circular cone with twice the
height of the pot. The volume V of a cone is proportional to "qr"i
i"r"ring
equal, insuring
equal lake votumes,lnd the side areas
equal icing areas'

3.2L Answers I ChuPter 5


Answers I Chapter 5 320
Reflect the triangle through the opposite side. The problem A little cube nestled in the corner of a big ore' 2' A big cube
€t7 now reduces to proving a diagonal of the resulting parrallelo- 37 1.
with a cubical chunk removed from one corncr' 3' Two cubes
gram is shorter than the sum of 2 adjacent sides. This follows meeting externally at a corner. If you perceived all 3, con-
from the triangle inequality. gratulaions! If ybu sa:w any other configuralions, what vere
they??

Each lattice line is either vertical with equation x : integer,


€i2 else it passes through (A, B) and (C, D) and has equation
or
y=
D-B BC_AD
C_A x + C _ A Since the coefficients are rational,
a8 Rotating the A about the square's center does not change the
area; what is lost in one quadrant is added to an ad-
"o**oiquadrant. Therefore, rotate so that the lwo legs of the A
jacent
y will be rational whenever x is. It follows, for example, that Lre flush.with the square's diagonals. Then the common area
the point (yr, \/Z ) does not lie on any lattice line. Hence the is readily seen to be lq lhat of the square or L2'25'
lattice lines do not cover the plane.

In order for any point on the track to be 3 times as far from


38 sin (X + Y) - sin X * sin Y only if X + Y = 360". If
difference between the angles is
the
10', one must be 175" and the
39 one flag as the other, the track must be circular, with diameter
3/+ the distance between the flags. The track must, therefore,
other 185'. have diameter 1980 ft., so that its circumference is 1980 r ft'
or about 3.36 furlongs.

31 In the lake. It is simple to verify that a point is inside a closed


curve if and only if it requires an odd number of "crossings,, to
be outside. In this case the number is 3. 10 Three hummingbirds were sharing thb feeding station- with
cycles of 7, fi: and 13 minutes, iespectively' in the order in
which he first observed them.

35 Picture one of the equal sides as the base and let the other side
swing in a semicircle. Since the base is fixed, the area is largest
when the altitude is longest, i.e., when the two equal sides are 11 A1l 4 triangles are similar giving two equations in A and B' A2
* 82 : 3dand (30-A) :B (20-B)' Solve for A (= 2'8677)'
perpendicular. Hence the third side will be a/ Z times the length then B. The desired length : @ - 307/s
of one of the equal sides. feet.

36 At zero distancel Regardless of the angles or lengths of the


sides, if a line is drawn connecting the two points of intersec-
tion, it will be perpendicular to the third side. (Recall that an
angle inscribed in a semi-circle is a right angle.) Hence the
point of intersection will lie on the third side.

Answers I Chapter 5 322 323 Answets I ChoPter 5


+ 1)
Let N be the smallest integer. The product is then N(N
Attsefieys (N + 2) (N + 3) : lN', +,N) (N: + 3N +since=2 2)
CHAPTER 6
iN, + 3N + 1)2 - t-'rtris is not a perfect square
positive squares cannot differ by 1'

The point values must be coprime; otherwise therl.1e **]]"'I


Each voyage involves the most ..restrictive', condition of the
*uny i-potsible scores. The number of impossibl:.tt::"^t-l'
(3'36)'
7 three, viz., no woman ever remains in the company of any man
then': Vz (x-1) (y-1) = 35- Hence x and y ate (2,71)'
(6,15), or (8,1i). (:,fe) and (6,15) are eljminated by the
unless her husband is present. The least .rr*6". -of couples n
coprimality condition, and since a score of 58 could be attained
in the first voyage is Z which also equals x and the numLer of
crossings is 5. If n: 3, x:2, then y: 11. If n: 4 or 5,
by making 29 tvto point free throws, the value of a free throw
is 8 and that of a field goal is 11.
x : 3, then y : 9, and y = 11, respectively. Happily however
forn)5,x:4andy=2n-1.
1,2,3- x + y + z= xYz. So A*!+-l--1.rfx<y
' xy
yz' xz
With an aggregate score of 28 points, there must have been 4 {: l-= L a, .o 1=tandxy<3.
events at_ 7 points per event. The only combination of place
z then
xY-Yz und xz xy-
scores which satisfies the requirements is 4 points for first
place, 2 for. second place, 1 for third place. In the high jump,
B won 4 points, A won 2, C won 1. In all other eventq A wdn IfweletthewidthsofthethreefarmsbeA,B,andC'wehavo
4 points, C won 2, B won l, consequently A won the pole vault. A (A + 8) = B (B + 28) : C (C + 34)' The equationA
(A + 8) - B (B * 28) has solutions A: 8' B:4 u"g l.:
yielding
aO, S : :2. Only the latter satisfies the third equation'
: and
the value C 30. The farms are, therefore, 4Ox48' 32x6O
x : 15, y : 20, z : 1.2. The
The simplest solution is where
3Ox64, all containing l92O sq. mi'
general formulais: x: ma n4;y:2mn (m2 + nz);z:2mn
(m2
-
- nzi"
361 tons.

Of all combinations of three factors whose product is 1296 only


are:
is 1,466,640 square feet. Rectangular dirnensions
two add up to the same street number. Th"se are lXlgXTZ
and 2x8X8l, both of whose factors sum to 91. Maynard needed
to make his second query only to resolve this ambiguity. Bar_
10 Area
1080 by 1358; 116a by 1260; and 970 by 1512 feet'

ring the possibility of a very precocious 6 year otO, ttre ages


dirnes and
must be l, 18, and 72. Letting N, D and Q stand for the number of tickels'
17 quarteis respectively, we have N + D + Q= 47' 5N + 10D

Ansuters I Chapter 6 324


325 Answars I Chapter 6
+ 25Q : 795 from which D + 4e : ll2. D must be of the than the other the wheels would overlap and could not be in
form 4k, Q must be of the form 2g _ k, so that
N is of the the same plane. Therefore, one wheel is 5 in' larger in radius
form 19 - 3k. Only D : 20, e:23, N : 4 satisfies
the than the other.
triangle inequality.

12 P + p * H : I00, and p + loD + 50H - 500, giving gD *


49H : 400. Since H < 10, the only value for *fri"h-+OO _
17 Connect the 3 midpoints forming 4 smaller equilateral triangles
with 4Vz" sides. At least 2 of the 5 points lie in the same tri-
angle; hence the maximum value of d is 4Vz",obtained by
49H is-divisible by 9 is H : l. Hence the bank contained of the 3 vertices and 3 midpoints-
1 choosing any 5
half dollar, 39 dimes, and 60 pennies.

(H + 10A) (A + l0H) : 1001 T + 100 H + 10A,


73 If the number of staff equals x, then each annual
the consultant 300 + 75 (x
-
bonus costs
t) dollars. Then (75x * 225)n
7a Since
1001 T- (A + 10H) (H + 10A - 10). The faclors of 1001,
namely 7, lt, and 13 must divide the right member, and only
= 6,000, where n : number of years. The only solution meet_ the value T : 6 permits an integral solution. Hence THAT is
ing the required conditions is n : g, x : 7.
6786.

11 /
If we assume the original number
rot
was a.b, we have ab _
r'9 The number of ways N obiects can be distributed arnong k urns
is kN. Since BG : GB and G 7 B, the only integer solution is
\"* O/ = 14.6. Sinceaandbmustbeintegers, bmustbe
4 girls and 2 boys.
4, and from 4a - a .4 : 14.6, we have a :
-
the original number was 5.4, written as 5 .
5. Therefore,
4 and evaluate d as ZO.

20 There is only one feasible answer. Junior is 36 rnd Dad 72'


Their respeciit" ug.t on the eight previous birthdays were: (1,
15 If we call the digits A, B and C, we have A + l0 B + 100 C
: C * R B + R2A or99C : (R2_ I) A + (R_
10) B.
37), (2,38), (3, 39), (4,40), (6, 4?.), (9, 45>, (12, 48),
(18,54).
R must be at least 3 in order to produce three distinct
Values of R from 3 to 13 yield no solution in
Jigil.
integers A]B,
C, all distinct and less than R (or 10, whichever i."r*ufllJ.
But R : 14 yields the solution A : 4, B 3, C : g. There_
foreR:14.834s-438a. =

l6 Let the radius of the smaller wheel be A and of the


wheel B. Then the length of the belt not in
wheels
larger
contact with
2y'-16tr--(ElJz. For this to be integral, (B_A)2
is
the
must be 25 or 144. If one wheel were 12 in.
larger in'raAlus

lnswers I Chapter 6 326


i
Answers I ChaPtet 6
f
I 327
t
.se&
There are no primes in this series. Consider the first nine num-
Am,su)era bers, the highest of which is 987,654,321. Eliminating even
numbers and the .five ending, leaves those ending io 9,7, 3, and
CIIAPTER 7 1. But the digital sum of each is either 9 or 6 making them all
divisible by 3. Next, any number higher than this number will
repeat the same digital sums since the digital sum o{ this ninth
number is 9.

If X : the least number of marchers, then X * 1 is congruent


to 0 modulo 3, modulo 5, modulo 7, and modulo 11. That is The largest possible number is 332. 22.
X+ l must be divisible by 3,5,7 and 11. The least number
which is so divisible is the product 3x5x7x1r since these four
numbers are prime factors. Hence the least number of marchers
We shall assume that the base of the number system is equal to
is 1154.
the number of fingers. If b is the base then we can write the
equation as follows:
5x2 - 5bx+ (b2 + 2b + 5) : 0. Thus b : 5 * 8 : 13 and
The remainders obtained when successive powers of 5 are di_ the Martians had 13 flngers.
vided by 7 form a repeating series: the ggb,gggtlr term of the
series is 6.

10 Denoting width and length a and b we obtain the equation


a: 2b + 4/b - 2, yielding 2 integral solutions: 3 X 10 or
fhe price was figured by adding the square of the sum of the 4x6.
digits of the previous price TO the previous price.

General solution: , = (, . *) and b: (, **)".'


As we have to exclude complex and improper fractions and 77
recurring decimals, the simplest solution is this: jg + 5
and 84 + 2/6. Both equal S4 1/3.
1,/3
One solution is therefore:
"
: 1 andb:
27
g.

A quick calculation of all the atoms results in a fisure with


9'"
digits numbered in the hundreds. As the total digits Jf 9,0 u."
in the hundreds of millions, the number of atoris is only an
12 The highest possible number is 81; therefore, thc root is less
than 300 but it can't be over 200 because as a palindrome it
would not be a prime. Being in the 100's the final square of the
infinitesimal fraction of it. number can only be 01 or 81. Only 5 or 9 as a central digit will
yield these. Therefore: 36481 : 1912.

1O0 miles, since the odometer reading now must be x 9g99.9


where x is any digit from 0 to 9.
13 My house number is 204, and there are 288 houses in rny road.

Answers I Chapter ? 328 329 Answers I Ch.apter 7


,d4&
11 P=2,i:2- Express one million in the scale of Ll (62335L>. The digits
79 indicate the number of prize winners of each amount. For ex-
ample, one received $1, five received $11, etc. There was a
Y,X:D+Y+ total of 20 winners. Since the number of winners of any given
75 If D is the number of the day in year

t?lD -t#l *[ffi]


amount was less than 11, the uniqueness of representation of
,-.0 7) is the day of the an integer in any scale guarantees that there ane no other
solutions.
week : :
falls' on, (Sun. 1, Mon. 2, etc.). If D - 1,
Y : 10ON + 1, this becomes X= 124 N+2+
iil pO In any base, 2 : fir.#.#:4.Irence fi: t. If B is
: 5N + , + []l (mod 7). This congruence has only the
L4) the base, 4 B + 1 = 3 (B + 4), and B = 11. (The Pue-
solutions O,2,3, and.5" Therefore the first day of a century can vigians use 1 1 as their base, since they have 5 fingers on one
fall only on Mon., Tues., Thurs., or Sat. The 21st century will hand and 6 on the other.)
start on a Monday.

Only for n : 1 or 3. Otherwise, the terminal digit of


t)'
21 while a square must end in 0, 1, 4,5,6, or 9.
! is 3,

16 since (n , - (l)': n3, it rollows that A = (t3o) :


31e,6oo; u : (,?r) : 318,8o1 is a solution. 5) 5) The extreme digits can only be 2 and 8 or 1 and 9 with respec-
fiJ TJ tive multipliers 4 and 9. In both cases there is no carry to the
first (leftmost) digit, and the second digit of the smaller number
is quickly found. It is 0 or I in the flrst case ald O in the
in the scale of 5 can end only in 0, 1 or 4. In
17 Squares the
scale of 10, a 0 must be preceded by a 0, and 1 or 4 must be
preceded by an even number. These even numbers in the pen-
second. Now working at the other end, one arrives at 4 X ?1978
: 87912 as the only solution satisfying the distinct digits con-
dition.
altimate position must be 0, 2 or 4, since 6 and 8 would be
impossible in the scale of 5" Proceeding in this way, imposing
similar restrictions on the other digits, we find 232324 as the
only number which is a square in both bases. 232324 : 3322
in the scale of 5 and = 4822 in the scale of 10.
23 A final zero can result only from the product of a 2 and a 5.
Since there will be more 2's than 5's in N!, the number of final.
zeros will be the highest power of 5 in N!, which is 10 for 49!
and 12 for 50!. Since 25!, 30!, 35!, and 40! have, respectively,
6, 7, 8, and 9 as the highest contained power of 5, the re-

r8 The first eight integers can be partitioned into only


3 47 8 , | 3 6I
be added to any
- 2 4 5 7 or
| 256-
14 5 8 - 2 3 67. Nine cannot
of these groups without forming an arithmetic
quired number is I 1.

progression.
24 In the factorization of a perfect nth power, each prime factor
has an exponent divisible by n. Consequently, if a lumber is a

Anstoers I Chapter 7 330 331 Answers I Chapter 7


L0482 - 9804 : 678. Decoding 7O243 with respect to each
square, cube, and a fifth power, every prime factor has an ex- solution, plus a little inference, yields the two riddle answers:
ponettt divisible by LCM (2, 3, 5) : 30' Thus the number is EARLY and CARTS.
i perfect 30th power, and the second smallest such number is
230"

29 C cannot be 4 since X and L would both be 2. Hence C = 9


and X = 3, and the unique solution: 453 + 485 = 938 is
25 The number of missing pages, P, is obviously even, while n,
the flrst missing page number, is odd (since it is a right hand
page). The number of the last page is n + p - l, and the total
obtained.

of the missing page numbers is


(n * -
2
p * p) _(n -_l)(n) :\pn+ p _ 1) : 8656
1)(n
2 Z'
30 The row totals are all either 4,9, or 16. But since the total of
t}:re 2l circles is 3(28) : 84, and no partition of 84 into 7
numbers of the form 4, 9, or 16 can contain a 4 (readily
or p (2n * p - l) : (32) (541) with the first factor even, the proven), there are four 9's and three 16's" Obviously the even
second odd. Hence p 255 and pp 255-286 are missing.
-- 32, n = rows are the ones which total 16, and the unique sequence
4 7 3 6 2 5 1 is obtained.

26 Let 1, a, b,
and suppose
T
. , n be the divisors of n in increasing order,
1
****
+ I
1

n -
2. Multiplying
1 He did. Since 499) \f@W it follows that 49,999 has no
non-trivial prime factors>499 (since it has none smaller). Also,
499 is obviously not a factor of 49,999. Hence 49,999 is prime,
through by n, we have n * * t + . * 1 : 2n ot
* and the factorization is complete.
nn n, where the left side consists of the
a- b
I
'
* 1 =
proper divisors of n in decreasing order. Ey definition n is
"perfect," the next two perfect numbers after 6 being 28 and
€t2 The only orthonym in English is TWENTY NINE. Polyglots
are invited to find orthonyms in other languages.
496"

it3 13! : L-2.3.4.5. . . 10.. . 13. Since five is a lactorof multi-


plicity two, 13! must enO in two zeroes. The flrst nurnber is,
27 Number the wives in base 3, thus: 1,2, 10, 11, 1'2,20, - ,
and note that if the last digit is not 2, the wife has only one
ring. If the last 2 digits are OZ or 22, she has two rings; if the
therefore, the right one. Alternate solution: the 2nd and 3rd
numbers are not divisible by 7.
lasf 3 digits are 012 or 212, she has three, etc. Hence the wife
withten rings had position I 1 I 1 1 I 1 1 2 (base 3) and the
sultan had 9,842 wives, (roughly 1 a day fot 27 years)- it1 Clearly O : 1, and ! : 0, and since there is obviously a carry
from column 2, 1 + I + n : the base. Also I l- A = [ and
E + A = 1 -l- the base. It follows that I = 4, A : 3, and
the base is six. It is, therefore, a reasonable inference that the
1, A : 0, and H : 9, leading
2A It can be quickly seen that W =
without much efiort to two solutions: 10512 - 9705 -- 867 and
visitors had three fingers on each hand.

333 Ansuters I ChaPter /


Answers I Chapter 7 332
85 If we let X : the number in the center square, the sum of the
numbers in any line is 3X. We seek, therefore, among the
Am,seuera
numbers 15, 18, 21, . .. 3X, . . . 99 one which when revlrsed
: X or X -+- i, where i equals 1,2,3 or 4. We find X : 1.7, CHAPTER B
X : i : 15 and X - Z4,X + i : 2'1. Hence the required
number is 17, and the magic square is, up to single motions
given below:

If the height is h, after the first drop it rises %h, next (V+)h,
etc. The distance up and down then is twice (Vz)h * (V+ )h
+ . . . The first drop is h feet, so the total is 3h or 30 feet.

The solution depends on maximizing y : xll*. The value of x


to maximize y is e. Thus ell")zrrl". If both sides are raised
to the zre Power one gets e")re.

A: tT. Cl a 1;rrt where t is the compounding period in


€t years. Thus.A : $ e : $2"71+.

The given series is the product of the two series

(r' +... and 1 *


] * (+)' +...,whoserespectile
sums are z ana The answer is, therefore, 3.
).

The nth partial product ,' ; Using the


* * #
fact that each numerator is half the denominator of the next
factor, this expression simplifies to . (2, + 1.)
] * =
'#+,the limit of which is ctearly j u, ,, -, *

Answers I Chapter 7 334 335 Answers I Chapter B


Let x be a term far out in the sequence. The next two terms
If the original sheet has dimensions
I JL
L and W and x is the side
77 are approximately rx and r2x. By the defining equation, r2x =
40(L+W)- if the volume is to be a
of a square cutout, * - -J! 3x f lrx or r2 - 2r - 3 =0. Rejecting tle negative root,
r-a
maximum. For x to be integral, L and W must be proportional I
-
J.

to 8 and 5. The completed box, therefore, has dimensions in


the ratio 6:3:1.
7p Regardless of the base B, the decimal .12345 .

than .Z.Hence the limiting value ;(.2


. . is
which equals
always less

3.
**
After performing this interpolating and halving N times, the as B-> a, .72345 . . -->zero.
3 '2N + I
sum of the series will be As N --J oo, this ap-
2NJij. Let A : the number of passengers required to break even and
proaches the limit
3
,.
73 A * x the actual number on board. Then (A * x) (450 - 3x)
will be collected. By differentiation this is maximum when
* = E0,3. Then the number of passenge6 i,4J4, "u"t'
A of length l/y wit}a one end moving along the x gr+:4
radius
(equator) and the other end at height y (latitude)
axis
generates paying cents. But 9i4 : , ({#4) :
a curve in which K : 2y, based on the equation dx/dy
(A + x) giving 45O:3A * Zx.Recalling that x : ry,
: -J-.
\/r -y If "
the radius makes angle g with the x axis, yz
,,,
3
A : 150 and x : 0. Thus the boat owner managed to achieve
only his break-even point for a profit of zero.
- sin d, and the equation transforms to x = lVtti*' ,.,
:2 x :
J2 x:
each quadrant. Thus AB a trifle less than 1.2 miles.
11 The correct derivation is A* log A. If this equals x Ax-1,
a log a. Then a log a : 3, or a* : e3, and a - 2.8564.

Integral z squared dz / from one to the cube root of three /


Multiplied by cosine / of three pi over nine / is the log of the
cube root of e. 75 Maximum volume is obtained when the 4 cut-out squares have
sides 1/6 those of the original square, in which case V :
* "'.
Hence the tray plus remnants will sell for 3 ,' * {9 .roit.,
27-
70 It is well
variables
known that the general polynomial of nth degree in k

tu, (n J k) ,"rrrrr. Since, however (' * o) =


_-
while the sheet wilt sell for S2 units. Equating these prices,
S : exactly one foot.
\n / \n /
(- il ") , there is symmetry between degree and number of for (3,5), all prime pairs are of the form (6N-1,6N
variables. In the specific case considered the number
is 8,008 in both instances"
of terms 76 Except
+ 1) with product 36 N2-1.
residue modulo 9, we note that 36N2-1
The digital sum being sirnply the
: -"1.:.8 (mod 9)-

)Jl Answers I ChaPter B


Answers I Chapter B 336

You might also like